FM ITE 21
137 A previously healthy 38-year-old female with a normal BMI has a few warts at the base of her great toe on the plantar surface. She noticed them a few years after she started to run regularly for exercise but has not tried any at-home treatments. They are minimally raised and rarely painful but occasionally cause irritation, and she would like to get rid of them. Which one of the following would be the most appropriate treatment? A) Over-the-counter salicylic acid B) Candida injections C) Application of duct tape D) Manual paring and extraction E) Laser treatment
A. A Cochrane review reports good evidence that salicylic acid is effective for the treatment of plantar warts. Candida injections may be indicated for warts that are difficult to treat, but they are not considered first-line treatments. The application of duct tape has not been shown to be more effective than placebo. Manual paring and extraction of plantar warts carry a greater risk for complications and are not necessary for these flat, minimally bothersome warts. Laser treatment may be effective, but the cost is not justified as an initial therapy in this simple case.
52 A 79-year-old male presents to your primary care clinic as a new patient. He is unaccompanied and tells you that he has no acute concerns and takes no regular medications, but just "needed a doctor." He is a retired lawyer and states that he lives alone. He has no significant past medical history but you are able to access his electronic medical records and note that he has a history of diabetes mellitus and hypertension, and has been to the local emergency department (ED) twice in the last 4 months for headaches. A workup from the ED was notable only for moderately elevated blood pressure. When asked, he does not appear to recall these visits. His vital signs are notable only for a blood pressure of 165/90 mm Hg. A physical examination reveals a thin but well appearing older adult who is mildly disheveled in appearance, with no other abnormalities. Which one of the following would be the most appropriate next
A. A cognitive assessment is indicated for this patient presenting with potential signs of self-neglect and memory lapses.
114 An asymptomatic 55-year-old female sees you for the first time. She asks for advice about laboratory results from a health maintenance examination performed last month by her previous physician. These results included a TSH level of 0.2 U/mL (N 0.4-4.2). Other laboratory results, including free T4 and free T3 levels, were normal. Her past medical history and a physical examination today, including a thyroid examination, are unremarkable. Which one of the following would you recommend at this time? A) Observation only B) A thyroglobulin level C) Thyroid antibody studies D) Thyroid ultrasonography E) A thyroid uptake scan with radioactive iodine
A. Approximately 1%-2% of adults will have subclinical hyperthyroidism with a low TSH level and normal T3 and T4 levels. The American Thyroid Association recommends observation for asymptomatic patients with mildly low TSH (0.1-0.4 U/mL), so further evaluation is not indicated in this patient. A thyroglobulin level might be helpful in differentiating exogenous thyroid intake from thyroiditis but would not be appropriate in this case. Further evaluation including thyroid antibody studies and a thyroid uptake scan with radioactive iodine should be considered in older patients (>65 years), those with very low TSH (<0.1 U/mL), and those with comorbidities. Thyroid ultrasonography is not indicated for abnormal thyroid function tests in asymptomatic patients with normal examinations.
1 A 67-year-old male sees you for a Medicare annual wellness visit. He tells you that his best friend had a stroke and he asks about his risk for stroke. He has no history of stroke, TIA, or neurologic symptoms. He has a family history of cardiovascular disease in his father, who had a myocardial infarction at age 65 and died from a thrombotic stroke at age 71. The patient exercises regularly and has a BMI of 27 kg/m2. His only current medical condition is hyperlipidemia, and his cholesterol level is at goal on rosuvastatin (Crestor), 10 mg daily. He also takes aspirin, 81 mg daily. His blood pressure is 125/78 mm Hg. Based on U.S. Preventive Services Task Force guidelines, which one of the following would be most appropriate at this time? A) No additional testing for stroke risk B) Auscultation for carotid bruits C) Carotid duplex ultrasonography D) Magnetic resonance angiography E) CT angiography of the carotid art
A. Carotid artery disease affects extracranial carotid arteries and is caused by atherosclerosis. This patient is asymptomatic and has no history of an ischemic stroke, neurologic symptoms referable to the carotid arteries such as amaurosis fugax, or TIA. He has risk factors for cardiovascular disease (age, male sex, hyperlipidemia), but the U.S. Preventive Services Task Force recommends against specific screening for asymptomatic carotid artery stenosis (D recommendation), which has a low prevalence in the general adult population. Stroke is a leading cause of disability and death in the United States, but asymptomatic carotid artery stenosis causes a relatively small proportion of strokes. Auscultation of the carotid arteries for bruits has been found to have poor accuracy for detecting carotid stenosis and is not a reasonable screening approach. Appropriate modalities for detecting carotid stenosis include carotid duplex ultrasonography, magnetic resonance angiography, and computed tomography, but these are not recommended for screening asymptomatic patients. Ref: Final Recommendation Statement: Screening for Asymptomatic Carotid Artery Stenosis. US Preventive Services Task Force, 2021.
159 An 85-year-old male with hypertension and coronary artery disease comes to your office for a routine wellness visit. He is accompanied by his wife, who notes that the patient's memory has been worsening over the last few years. His current medications include carvedilol (Coreg), losartan (Cozaar), hydrochlorothiazide, and aspirin. A brief screening test is positive for cognitive impairment. The Saint Louis University Mental Status (SLUMS) examination places the patient in the dementia category. You order further testing, including a TSH level, a CBC, a comprehensive metabolic panel, and a vitamin B12 level. Which one of the following should also be included in the workup? A) The Geriatric Depression Scale B) Apolipoprotein E epsilon 4 allele testing C) An EEG D) A PET scan E) Cerebrospinal fluid analysis
A. Depression in the elderly can cause symptoms similar to those of dementia. Also, many patients with dementia concurrently have depression. It is recommended that depression be treated first if found (SOR C). If cognitive symptoms improve with depression treatment, pseudodementia is diagnosed. The recommended workup for dementia includes a TSH level, a CBC, a comprehensive metabolic panel, and a vitamin B12 level; depression screening; and noncontrast MRI of the brain. MRI of the brain is recommended to rule out stroke, mass, or hydrocephalus. If MRI cannot be performed, then CT is indicated. Testing for the apolipoprotein E epsilon 4 allele is not a diagnostic test for Alzheimer's dementia. It can be ordered for children of affected individuals to assess the risk of developing the disease. An EEG would be useful if the patient also experienced seizures, but it is not routinely indicated. A PET scan is not appropriate in the evaluation for dementia. Cerebrospinal fluid (CSF) testing is indicated for patients with rapidly progressive symptoms of dementia. Testing for infection and prior disease can be accomplished through CSF analysis.
24 A 4-month-old female is brought to your office by her mother for a well child visit. An examination reveals slightly rough erythematous patches on both cheeks and her chin, as well as on her thighs. Which one of the following would be most appropriate at this time? A) Twice-daily application of a fragrance-free moisturizer with a high lipid-to-water ratio B) Twice-daily application of a low-potency topical corticosteroid C) Application of a topical calcineurin inhibitor for the facial lesions and a low-potency topical corticosteroid for the other areas D) Allergy testing E) Referral to a dermatologist
A. For atopic dermatitis in an infant, the most appropriate initial treatment is twice-daily application of a fragrance-free moisturizer with a high lipid-to-water ratio.
150 A 50yo M has acute URI w/cough that has improved but not resolved. He presents with a 2-day history of chest pain that began gradually. The pain is worse when supine, takes deep breaths, or coughs, and he says the pain is relieved when he leans forward while sitting. He is afebrile, and his vital signs are normal. An EKG shows acute pericarditis, a normal troponin level, and he does not appear ill. 7 days later, he tells you that the pain is improved. He's afebrile, and vital signs are normal. On exam, he still has a pericardial friction rub but no murmurs, and his lungs are clear. Which one of the following would be most appropriate at this time? A) Continue his medications and add colchicine (Colcrys) B) Continue his medications and add prednisone C) Discontinue his medications and start aspirin D) Discontinue his medications and start indomethacin E) Admit him to the hospital and consult a cardiologist
A. For ongoing pericarditis, adding colchicine to current medications, typically NSAIDs, is the most appropriate step to further manage symptoms.
63 A 55-year-old female with type 2 diabetes sees you because of early satiety, nausea, vomiting, bloating, and postprandial fullness that is sometimes accompanied by upper abdominal pain. Since these symptoms have developed she has also noted increasing difficulty with blood glucose control. Which one of the following would be the best study for confirming the most likely diagnosis? A) Gastric emptying scintigraphy with a solid meal B) Hepatobiliary scintigraphy (HIDA) C) An upper gastrointestinal series with small-bowel follow-through D) Abdominal ultrasonography E) Abdominal CT
A. Gastroparesis is a complication of diabetes mellitus, and gastric emptying scintigraphy with a solid meal is the first-line study for confirming the diagnosis.
21 A 30-year-old male is diagnosed with hereditary hemochromatosis. Periodic therapeutic phlebotomy may be appropriate to prevent A) chronic liver disease B) chronic renal disease C) encephalopathy D) myelofibrosis E) Wilson disease
A. Hereditary hemochromatosis leads to iron deposits in the liver, which may result in chronic liver disease and hepatocellular cancer. Periodic therapeutic phlebotomy is indicated to prevent chronic liver disease.
109. A 58-year-old male sees you for evaluation of left ankle pain after he slipped on some ice in his driveway last night. He felt immediate pain over the lateral ankle, which started swelling over the next hour. He elevated his foot, applied ice, and took ibuprofen. This morning the ankle remains swollen and also appears bruised. He is able to walk on it with some pain. On examination you note typical findings of an ankle sprain. He asks you for medication to manage his pain. Which one of the following medications has the best evidence for providing pain relief while also minimizing side effects? A) Topical diclofenac gel (Pennsaid) B) Topical menthol gel C) Oral hydrocodone/acetaminophen (Lortab) D) Oral ibuprofen E) Oral tramadol
A. In 2020, the American College of Physicians and the American Academy of Family Physicians published a guideline regarding the treatment of acute pain from musculoskeletal injuries. This systematic review found good evidence to support the recommendation that topical NSAIDs be used as first-line therapy to reduce pain and improve physical function. Topical NSAIDs were the only intervention that improved multiple outcomes and were not associated with a statistically significant increase in the risk for adverse events.
123 A 5-year-old female has acute, severe bacterial sinusitis. Which one of the following would be most appropriate for this patient? A) Amoxicillin/clavulanate (Augmentin) B) Cephalexin C) Clarithromycin (Biaxin) D) Doxycycline E) Levofloxacin
A. Patients with acute bacterial sinusitis who do not improve while taking the usual dose of amoxicillin, who have recently been treated with an antimicrobial (within the past 90 days), who have an illness that is moderate or more severe, or who attend day care should be treated with high-dose amoxicillin/clavulanate in two divided doses. Alternate therapies include cefdinir, cefuroxime, or cefpodoxime. A single dose of ceftriaxone, 50 mg/kg daily, either intravenously or intramuscularly, can be used in children who are vomiting. Once there is clinical improvement, usually within about 24 hours, an oral antibiotic can be started. Cephalexin is not recommended for treating acute bacterial sinusitis because of inadequate antimicrobial coverage of the major organisms. Clarithromycin is not recommended as empiric therapy because of high rates of resistance in Streptococcus pneumoniae. The use of doxycycline is not appropriate in children. Levofloxacin would be appropriate if the patient had a history of type I hypersensitivity to penicillin.
90 A 13-year-old male is brought to your office for evaluation of back pain. Plain radiography would be indicated at this time if the patient has A) pain that awakens him at night B) pain that is localized to the midthoracic spine C) pain that is increased with flexion D) intermittent pain that has persisted for 2 weeks E) a recent history of an upper respiratory infection
A. Plain radiography would be indicated if the patient has: Pain that is localized to the midthoracic spine - Guidelines suggest that imaging in children with back pain can be delayed unless there are abnormal neurologic findings, pain that awakens the patient at night, or pain that radiates or persists for more than 4 weeks. However, localized pain to the midthoracic spine may warrant imaging at this time.
116 A 73-year-old female sees you because of a 2-month history of the gradual onset of bilateral swelling of several metacarpophalangeal joints with associated joint stiffness, which lasts for about 1 hour each morning. Her past medical history and an examination are otherwise unremarkable. Elevation of which one of the following would provide the most support for a specific diagnosis of her new arthritis symptoms? A) Anti-citrullinated protein antibody B) C-reactive protein C) Erythrocyte sedimentation rate D) Leukocyte count and differential E) Rheumatoid factor
A. Rheumatoid arthritis (RA) affects about 1% of people over their lifetime, with women being affected more often than men. RA can be diagnosed after considering the patient's medical history and physical examination and the results of serology and acute phase reactant tests. Some of these tests are often elevated in patients without RA, so family physicians need to know how to interpret positive laboratory results. This patient's history and the physical examination support a possible diagnosis of RA. Anti-citrullinated protein antibody is >95% specific for RA when significantly elevated. Acute phase reactants are very nonspecific and may be positive due to infection, other autoimmune conditions, age, or obesity. Leukocyte counts may be elevated because of infection, cancer, smoking, and other conditions. Rheumatoid factor is also nonspecific for RA and may be positive due to cancer, infection, and other autoimmune conditions.
65 A 60-year-old male who has type 2 diabetes comes to your office with an acute onset of fever, chills, and malaise. He says that he is feeling progressively worse. His temperature is 40°C (104°F). An examination reveals redness, tenderness, and swelling of the penis, scrotum, and perineal area. Which one of the following medications is most likely to cause this condition? A) Dapagliflozin (Farxiga) B) Exenatide (Byetta) C) Insulin glargine (Lantus) D) Pioglitazone (Actos) E) Sitagliptin (Januvia)
A. SGLT2 inhibitors (canagliflozin, dapagliflozin, empagliflozin, and ertugliflozin) are associated with a higher rate of genitourinary infections, including necrotizing fasciitis of the perineum (Fournier's gangrene). Dapagliflozin is most likely to cause this condition.
13 Which one of the following represents a CONTRAINDICATION to statin use? A) Current pregnancy B) Chronic hepatitis C infection C) End-stage renal disease D) Myositis associated with a creatine kinase level five times the upper limit of normal E) Transaminitis due to nonalcoholic steatohepatitis
A. Statins are contraindicated during pregnancy and not recommended during breastfeeding. Chronic hepatitis C infection, end-stage renal disease, myositis associated with a creatine kinase level up to 10 times the upper limit of normal, and transaminitis due to nonalcoholic fatty liver disease are not contraindications.
148 In patients diagnosed with COPD, testing should be considered for which one of the following underlying conditions? A) 1-Antitrypsin deficiency B) Cystic fibrosis C) Hemochromatosis D) Williams syndrome E) Wilson's disease
A. Testing for 1-antitrypsin deficiency should be considered in symptomatic COPD patients with fixed airflow obstruction. It's an important underlying condition to identify in COPD patients.
155 The American Thoracic Society/Infectious Diseases Society of America guidelines recommend which one of the following for the diagnosis and initial management of non-severe community-acquired pneumonia in adults? A) Use of a validated clinical prediction rule to determine the need for hospitalization B) Urine antigen testing for Legionella C) Blood and sputum cultures to guide therapy D) Procalcitonin to determine the need for antibacterial therapy E) Coverage for methicillin-resistant Staphylococcus aureus (MRSA)
A. The American Thoracic Society (ATS) and the Infectious Diseases Society of America (IDSA) guideline recommends the use of a validated clinical prediction rule, preferably the Pneumonia Severity Index (PSI), to determine the need for hospitalization in adults diagnosed with community-acquired pneumonia (CAP) (strong recommendation). The yield of blood cultures is around 2% (outpatients) to 9% (inpatients) in adults with non-severe CAP. A sputum culture and a Gram stain of respiratory secretions are recommended in patients classified as having severe CAP, or in those with strong risk factors for methicillin-resistant Staphylococcus aureus (MRSA) or Pseudomonas aeruginosa. Randomized trials have failed to show a benefit for urinary antigen testing for Streptococcus pneumoniae and Legionella. ATS/IDSA guidelines recommend empiric antibiotic therapy for adults with clinically suspected and radiographically confirmed CAP regardless of the initial serum procalcitonin level (strong recommendation). Coverage for MRSA is not recommended in patients without risk factors for MRSA pneumonia.
147 According to the Ottawa knee rule, a radiograph would be indicated for a patient presenting with an acute knee injury if the examination reveals tenderness to palpation over the A) fibular head B) lateral joint line C) medial joint line D) tibial tubercle E) upper medial aspect of the tibia
A. The Ottawa knee rule guides the need for radiographs in acute knee injuries. Tenderness to palpation over the fibular head is one of the criteria for imaging.
64 The administrator in your practice recently attended a quality improvement conference. He would like to start a clinic-wide program to focus on lead screening in asymptomatic children 5 years of age and younger. After reviewing U.S. Preventive Services Task Force guidelines, which one of the following should you tell the practice administrator? A) There is insufficient evidence to recommend for or against lead screening in children 5 years of age and younger B) All children 5 years of age and younger should undergo lead screening C) All children who live in housing built before 1978 should undergo lead screening D) There are accurate and reliable screening questionnaires to guide lead screening
A. The U.S. Preventive Services Task Force (USPSTF) has found adequate evidence that questionnaires and other clinical prediction tools to identify asymptomatic children with elevated blood lead levels are inaccurate. The USPSTF went on to conclude that the current evidence is insufficient to assess the balance of benefits and harms of screening for elevated blood lead levels in asymptomatic children 5 years of age and younger.
15 A 13-year-old baseball player who is right-hand dominant is brought to your office because of a 3-week history of pain in the right shoulder. He recalls no specific injury but has been pitching at least weekly for the past 3 months. He has moderate tenderness about the anterior and lateral deltoid. In addition to radiographic studies, the best initial management would include A) complete rest from throwing activities B) a sling and swathe C) sugar-tong splints both proximal and distal to the elbow D) physical therapy
A. The insidious onset of this patient's pain without known injury suggests an overuse injury. Pending radiograph results, the best initial management strategy is complete rest from throwing activities.
28 A 72-year-old female with a history of type 2 diabetes and hypertension presents to your clinic because of fatigue and depression for the last 5-6 months. She has gained about 7 kg (15 lb) and now has a BMI of 32 kg/m2. A repeat test 1 month later shows a TSH level of 7.4 U/mL and a free T4 level of 1.6 ng/dL (N 0.8-2.8). Treatment of this patient with L-thyroxine A) has no proven benefit B) can increase grip strength C) can increase her energy level D) can help improve depression symptoms E) can reduce her BMI
A. The management of subclinical hypothyroidism (elevated TSH with normal free T4) does not show proven benefits, including improvement in grip strength, energy level, or depression symptoms. Treatment does not reduce BMI.
82 A 70-year-old female comes to your office for a routine health maintenance examination. Her past medical history is notable for well controlled hypertension and right knee osteoarthritis, and she underwent total knee replacement 1 year ago with an uncomplicated postoperative course. She mentions that she has an appointment to have a tooth filled and asks for your advice on antibiotic prophylaxis for her upcoming dental procedure. Which one of the following would be the most appropriate recommendation regarding antibiotic prophylaxis for her? A) No antibiotic treatment B) A single dose of oral amoxicillin C) A 3-day course of oral amoxicillin D) A single dose of oral clindamycin (Cleocin) E) A single dose of intravenous cefazolin
A. The most appropriate recommendation is: No antibiotic treatment - Recent guidelines recommend against routine antibiotic prophylaxis for dental procedures in patients with a history of joint replacement, except for situations with increased infectious risk, such as immunocompromise or a history of previous joint infection.
78 A 70-year-old male is brought to your office by a family member. The patient is concerned about a tremor, which is most noticeable at rest and seems to get better with voluntary movements. On examination, his vital signs are unremarkable. He blinks infrequently during the examination, his face seems to be relatively immobile, and he feels rigid. He has some difficulty rising from a chair and walks with small, shuffling steps. The remainder of the neurologic examination is normal. You note some seborrhea of the scalp. A recent comprehensive laboratory evaluation is unremarkable. Which one of the following is necessary to confirm the diagnosis? A) No diagnostic testing B) CT of the brain C) MRI of the brain D) EEG E) A lumbar puncture
A. The prevalence of Parkinson's disease increases with age and shows a slight predominance toward males. Bradykinesia is a key diagnostic criterion. Muscular rigidity, resting tremor, and postural instability are other symptoms. Nonmotor symptoms such as depression, anxiety, fatigue, and insomnia are also common. Parkinson's disease is a clinical diagnosis and seldom requires testing. Imaging such as CT, MRI, or EEG can be useful in ruling out other diagnoses but will not reveal findings suggestive of Parkinson's disease. A lumbar puncture is not necessary to confirm Parkinson's disease.
5 A 45-year-old left hand-dominant female presents to your office with a lump on her hand. She first noticed the lump 2 weeks ago and thinks it has gotten bigger. She does not recall any injury. She has not had any numbness, weakness, or tingling. She has minimal discomfort when she presses on the lump, and it does not affect her activity. On examination her left wrist is neurovascularly intact. You note the volar wrist mass shown below. Which one of the following management options would you recommend? A) Re-examination if she develops numbness, weakness, or increased pain B) Immobilization of the wrist for 6 weeks and then re-examination C) Aspiration of the lesion D) Aspiration and injection of the lesion with a corticosteroid E) Referral for excision of the lesion
A. This patient has a ganglion cyst, which is common and resolves spontaneously in 50% of cases, and watchful waiting would be most appropriate at this time. Treatment is indicated if the cyst is causing significant symptoms such as pain, numbness, or weakness, or for cosmetic reasons. Aspiration of the lesion is the initial treatment, although recurrence may occur in 85% of cases. Immobilizing the wrist with a splint or brace is sometimes helpful in the short term if the patient is bothered by the symptoms, but immobilization does not provide lasting relief and could cause muscle atrophy. Corticosteroid injections have not shown any benefit. Referral for excision is appropriate if there has been no improvement. Patients should be advised that there is a 10%-15% recurrence rate even after excision. Ref: Armstrong AD, Hubbard MC (eds): Essentials of Musculoskeletal Care, ed 5. American Academy of Orthopaedic Surgeons, 2015, pp 510-516.
14 A 2-year-old female is brought to the urgent care clinic because of a fever. On examination, she has a temperature of 39.7°C (103.5°F). Within a short period of time while at the clinic, she develops a barking cough and respiratory distress, and you note rapid deterioration of her condition. Which one of the following is the most likely diagnosis? A) Bacterial tracheitis B) Epiglottitis C) Foreign body aspiration D) Peritonsillar abscess
A. This patient has bacterial tracheitis, which includes a high fever, barking cough, respiratory distress, and rapid deterioration. The clinical presentation is characteristic of bacterial tracheitis, which requires prompt attention. Epiglottitis, foreign body aspiration, and peritonsillar abscess have different clinical presentations.
50 A 19-year-old female presents with a 4-year history of intermittent facial acne. She tells you that her acne has never completely resolved, and it worsens during her menstrual period. She has tried various over-the-counter facial cleansers although she does not recall what they contained. On examination she has scattered open and closed comedones, and pustules on her forehead and around her mouth. Which one of the following treatments would you recommend? A) Topical adapalene (Differin) B) Topical clindamycin (Cleocin T) C) Oral doxycycline D) Oral isotretinoin (Absorica)
A. This patient has mild inflammatory acne as indicated by her combination of comedones and pustules. She does not have extensive skin involvement and should benefit from the use of a topical agent. Topical retinoids, including adapalene, tretinoin, tazarotene, and trifarotene, are appropriate for the treatment of mild to moderate acne as single agents, although they may be more effective when combined with a topical antibiotic or benzoyl peroxide. Topical antibiotics can lead to bacterial resistance and should not be used as monotherapy. Oral antibiotics are appropriate for the treatment of moderate to severe acne that has failed to respond to topical treatment. Oral isotretinoin is reserved for the treatment of severe nodular acne.
49 A 2-year-old female is brought to your office by her mother because of a cough and fever. The mother also tells you that the child has had a reduced appetite but she is drinking fluids normally. The child was born at term and has previously been healthy. On examination the child appears alert and happy. She has a temperature of 37.2°C (99.0°F), a pulse rate of 100 beats/min, a respiratory rate of 30/min, and an oxygen saturation of 98% on room air. An HEENT examination reveals clear rhinorrhea. Auscultation of the lungs reveals mild expiratory wheezing throughout with no crackles, and you note no signs of respiratory distress such as retractions or use of accessory muscles of respiration. Which one of the following would be the most appropriate next step? A) Reassurance only B) A nasal swab for respiratory syncytial virus C) A chest radiograph D) Nebulized albuterol E) Oral amoxicillin
A. This patient has symptoms typical for respiratory syncytial virus (RSV) bronchiolitis. Since the patient shows no signs of distress and is well hydrated, no specific treatment is necessary. Neither testing for RSV nor obtaining a chest radiograph would change management and therefore would not be indicated. Albuterol is ineffective for the wheezing associated with RSV since the mechanism of wheezing is not due to bronchospasm. Antibiotics are not indicated without evidence of a secondary bacterial infection.
122 A 38-year-old female presents with a 1-week history of pain in her left heel. The pain started abruptly while she was playing tennis and she was unable to finish the game. She has been applying ice, taking ibuprofen, and wearing an ankle brace without improvement. She walks with a limp. An examination is significant for localized tenderness over the left posterior leg approximately 3 cm above the calcaneus. The Thompson test (squeezing the calf) is significant for the absence of plantar flexion of the left foot. Her strength is decreased with plantar flexion of the left ankle. There is normal passive range of motion, as well as normal pulses and sensation in the left foot. Which one of the following is the most likely cause of her pain? A) Achilles tendon rupture B) Plantar fasciitis C) Proximal fifth metatarsal fracture D) Stress fracture of the fibula E) Syndesmosis injury (high ankle sprain)
A. This patient most likely has an Achilles tendon rupture based on the history and examination, with pain localized to the posterior leg 2-6 cm above the calcaneus. The Thompson test is positive when there is no plantar flexion of the foot with squeezing the calf, due to a disruption of the Achilles tendon. Plantar fasciitis typically involves heel pain that is worse with the first steps after a period of non-weight bearing and then improves with ambulation. Typically there is tenderness on the plantar surface of the foot along the plantar fascia. A proximal fifth metatarsal fracture would present with foot pain and a limp and there is typically point tenderness over the fracture site. A stress fracture may cause pain on palpation and a limp, but the Thompson test would be negative. A syndesmosis injury typically involves ankle pain, swelling, and instability. Pain is elicited at the site of the syndesmosis on the squeeze test, and external rotation of the foot and ankle typically reproduces the pain.
113 An otherwise healthy 46-year-old female presents with a 10-day history of recurring pain in the right cheek and gums. She says the pain feels like electric shocks lasting a few seconds and recurring "hundreds of times a day." She says that smiling and brushing her teeth can trigger the pain. She does not have a history of recent dental work, trauma, fever, or myalgia. She feels well aside from the facial pain. She has tried OTC analgesics without relief. There is no pain with dental percussion, and the skin and mucous membranes of the nose and mouth are unremarkable. Which one of the following would be the most appropriate treatment for this patient's condition? A) Carbamazepine (Tegretol), 200 mg twice daily B) Prednisone, 40 mg daily C) Sumatriptan (Imitrex), 6 mg subcutaneously D) Valacyclovir (Valtrex), 1000 mg three times daily E) High-flow oxygen for 20 minutes
A. Trigeminal neuralgia is a clinically diagnosed condition that is characterized by brief, sudden, unilateral pain in the distribution of one of the three branches of the trigeminal nerve. The pain, which is often triggered by minimal stimulus, is paroxysmal with episodes of remission that can last for months. First-line treatment for the condition is carbamazepine or oxcarbazepine (SOR C). Other suggested treatments include lamotrigine, baclofen, and surgical treatments, including microvascular decompression. Valacyclovir would be an appropriate treatment for herpes zoster, which can also cause unilateral electrical shock-like pain in the face. However, patients with herpes zoster typically would have developed the characteristic lesions by 10 days. Short-term corticosteroids are not recommended for trigeminal neuralgia and there is minimal evidence of their effectiveness for herpes zoster.
18 A 57-year-old male recently diagnosed with acute lymphoblastic leukemia presents to the emergency department with intractable nausea, vomiting, and myalgias. His first chemotherapy infusion was administered earlier in the day. Which one of the following electrolyte disturbances would be consistent with tumor lysis syndrome? A) Hypocalcemia B) Hypokalemia C) Hyponatremia D) Hypophosphatemia E) Hypouricemia
A. Tumor lysis syndrome is a common complication of chemotherapy in hematologic malignancies. Hyperphosphatemia, hyperkalemia, and hyperuricemia are indicative of tumor lysis syndrome. Calcium levels are decreased due to binding with free phosphorus and a depletion of calcium in the bloodstream.
55 A 62-year-old female sees you for a routine health maintenance visit and asks for your advice regarding vitamin D supplementation. She is healthy and active, and jogs 1-2 hours three times weekly. Which one of the following would be the most appropriate advice regarding vitamin D supplementation in this patient? A) It is not recommended because she is asymptomatic B) It will reduce the risk of certain cancers C) It will reduce the risk of depression D) It will reduce the risk of diabetes mellitus E) It will reduce the risk of fractures
A. Vitamin D supplementation is not recommended for asymptomatic individuals, as evidence is insufficient to support its use in reducing the risk of various conditions.
112 A 51-year-old female presents with concerns about a change in her cognition. She says she has difficulty retrieving words, loses her train of thought, and goes into a room and forgets why she came there. She also has had more frequent hot flashes and sleep disturbances. She still menstruates but has noticed a change from her previous pattern. A physical examination is unremarkable, and recent laboratory tests were all normal, including vitamin B12 and thyroid studies. Cognitive testing is normal. Which one of the following would be the most appropriate next step? A) Reassurance only B) CT of the head C) MRI of the brain D) Hormone therapy E) Referral to a neurologist
A. Women experience subjective cognitive difficulties during their menopausal transition. This may include retrieving numbers or words, losing one's train of thought, forgetting appointments, and forgetting the purpose of behavior such as entering a room. Clinical studies of these women showed intact cognitive test performance. The treatment consists of patient education and reassurance, since studies have shown that 62% of women report subjective cognitive problems during their menopausal transition. Imaging and referral to a neurologist are not indicated, and there are no trials that support the use of hormone therapy.
96 Question: A 58-year-old male with a history of a neurogenic bladder comes to your office as a new patient. He recently elected to have placement of a chronic indwelling urethral catheter and asks how to reduce his risk of urinary tract infections (UTIs). What is most effective for preventing UTIs in patients with chronic indwelling urethral catheters? A) Routine daily hygiene of the meatal surface with soap and water B) Daily periurethral cleaning with iodine C) Daily oral antibiotics based on prior urine culture sensitivities D) Routine instillation of an antimicrobial solution into the drainage bag E) Regularly scheduled catheter exchanges at fixed intervals
Answer: A Explanation: Routine daily hygiene of the meatal surface with soap and water is the most effective measure for preventing catheter-associated urinary tract infections (CAUTIs) in patients with chronic indwelling urethral catheters.
99 Question: A 39-year-old male with no significant past medical history presents to your office to establish care. He does not take any medications. A review of systems is negative. He has a BMI of 22 kg/m2 and a blood pressure of 141/82 mm Hg. What would be the most appropriate next step? A) Ambulatory blood pressure monitoring B) Recommending a weight loss of 5 lb C) Hydrochlorothiazide, 25 mg daily D) Lisinopril (Prinivil, Zestril), 10 mg daily E) Metoprolol tartrate (Lopressor), 25 mg twice daily
Answer: A Explanation: The most appropriate next step is ambulatory blood pressure monitoring to confirm the diagnosis of hypertension and screen for potential "white coat" hypertension.
93 Question: A 6-year-old female who recently moved to the United States from India requires a physical examination prior to entering the public school system. Her immunizations are up to date, including bacillus Calmette-Guérin vaccine at birth. Her family history is positive for her paternal grandfather being treated for latent tuberculosis infection. What is the most appropriate next step? A) An interferon-gamma release assay (IGRA, QuantiFERON-TB Gold) B) A nucleic acid amplification test C) A tuberculin skin test, and follow-up in 72 hours D) Three acid-fast bacilli sputum samples and N95 mask use E) A chest radiograph
Answer: A Explanation: The most appropriate next step is to perform an interferon-gamma release assay (IGRA) due to the patient's positive family history and prior bacillus Calmette-Guérin (BCG) vaccination, which can cause false positives with tuberculin skin tests.
95 Question: A 6-year-old female is brought to your office by her parents for a routine well child visit. Her height is 121 cm (48 in), which is the 91st percentile for her age, and her weight is 27 kg (59 lb), which is the 92nd percentile for her age. What would be an appropriate recommendation regarding vehicular safety for this child? A) She should use a rear-facing car seat B) She may use either a booster seat or a car seat C) She can safely wear a seatbelt without a booster seat when she reaches 127 cm (50 in) D) She should not use a car seat because her height exceeds the limit of common car seats E) She may sit in the front seat once she turns 11 years of age
Answer: B Explanation: Children 4-8 years of age may use either a booster seat or a car seat. This child is too large for rear-facing seats and should use appropriate restraints based on her size.
91 Question: The U.S. Preventive Services Task Force recommends one-time screening for abdominal aortic aneurysm in A) men 55-75 years of age who have ever smoked B) men 65-75 years of age who have ever smoked C) men and women 65-75 years of age who have ever smoked D) men and women 65-75 years of age who currently smoke
Answer: B Explanation: The U.S. Preventive Services Task Force (USPSTF) recommends one-time screening for abdominal aortic aneurysm (AAA) in 65- to 75-year-old men who have ever smoked. The primary method of screening is conventional abdominal duplex ultrasonography.
94 Question: A 30-year-old female presents with episodes of severe vertigo lasting 4-5 hours and associated with tinnitus, nausea, and vomiting. On examination, the Dix-Hallpike maneuver is negative. What is the most likely diagnosis? A) Benign paroxysmal positional vertigo B) Meniere's disease C) Multiple sclerosis D) Vestibular migraines E) Vestibular schwannoma
Answer: B Explanation: The patient's symptoms of episodic vertigo, hearing loss, and tinnitus are consistent with Meniere's disease.
98 Question: A 63-year-old female sees you for follow-up after an emergency department visit. CT confirmed diverticulitis, and she was treated as an outpatient. The discharge paperwork indicates she has a left superior adrenal nodule measuring 1.2 cm. Which concurrent condition should prompt a hormonal workup? A) Hyperlipidemia B) Hypertension C) Renal cell carcinoma D) Rheumatoid arthritis E) Type 2 diabetes
Answer: B Explanation: The presence of a left superior adrenal nodule should prompt a hormonal workup in the presence of hypertension, as hyperfunctioning adrenal adenomas can cause this condition.
100 Question: A 4-month-old female is brought to your office by her parents as a new patient for a well child visit. The parents have refused all vaccinations other than hepatitis B at birth. Which communication strategy is recommended to promote vaccine acceptance? A) Emphasize the benefits and downplay the risks of vaccines B) Clarify specific concerns and provide factual information C) Avoid future discussions with the family about vaccines D) Dismiss the family from the practice
Answer: B Explanation: To promote vaccine acceptance, it is recommended to clarify specific concerns and provide factual information. Building trust and addressing concerns are crucial strategies. Dismissing the family from the practice is not recommended.
97 Question: A 68-year-old male presents with a burn on his lower leg after trying to light a bonfire with kerosene. Examination reveals blistering and a denuded central area that does not blanch with pressure. What is the proper classification of this burn? A) Superficial burn B) Superficial partial-thickness burn C) Deep partial-thickness burn D) Full-thickness burn
Answer: C Explanation: The burn is classified as a deep partial-thickness burn, as it blisters but does not blanch with pressure.
154 A 55yo F presents with swelling and some redness to her right ankle with gradual onset over 1 week. She denies injury, fever, systemic illness, or pain. Her pmh iincludes DM2 with polyneuropathy that is controlled, HTN, HLD, and a BMI of 35 kg/m2. On examination, her right ankle and foot are slightly larger than the left, exhibit faint erythema, and feel slightly warmer than the left. No pain is noted with palpation, and her ankle ligaments appear to be intact. Pedal pulses are 2+ bilaterally, and she has no calf pain or swelling. Which one of the following would be the most appropriate next step? A) Reassure her that the lack of pain indicates the absence of a serious disease process B) Prescribe antibiotics for presumed cellulitis C) Recommend compression stockings, leg elevation, and monitoring D) Provide an ankle stabilizing brace E) Obtain bilateral weight-bearing foot radiographs
E. Acute Charcot neuropathy is a commonly missed diagnosis, and the diagnosis is delayed in up to 25% of cases. The diagnosis should be considered in patients over age 40 with neuropathy and obesity who present with unilateral foot swelling. There may be associated erythema and warmth, and pain may be absent. In a patient with suspected acute Charcot neuropathy, bilateral weight-bearing radiographs are recommended to detect fractures of the midfoot. Acute Charcot neuropathy is frequently painless, and its consequences can be severe, so it would be inappropriate to counsel a patient that lack of pain means the absence of serious disease. Charcot neuropathy is commonly misdiagnosed as cellulitis. In this patient's presentation, cellulitis is not a clear diagnosis, and Charcot neuropathy needs to be considered before initiating treatment for cellulitis. Compression stockings and leg elevation are appropriate for peripheral edema when other causes of edema have been evaluated and addressed, but in this case the swelling is lower on the leg than what compression stockings would usually treat, and further evaluation is required prior to treatment. There is no evidence for ankle sprain or instability in this patient, so an ankle brace would not be appropriate.
71 Which one of the following is the preferred method of diagnosing lymphoma in a 60-year-old male who presents with weight loss, unexplained fever, and axillary adenopathy? A) CT of the chest, pelvis, and abdomen pre- and post-contrast B) A PET-CT scan C) A bone scan D) A bone marrow aspiration and biopsy E) An open lymph node biopsy
E. An open lymph node biopsy is the preferred method for making the diagnosis of lymphoma. Although fine-needle aspiration and core needle biopsy are often part of the initial evaluation of any adenopathy, neither will provide adequate tissue for the diagnosis of lymphoma. A PET-CT scan may be used for staging. A bone scan or CT alone is not part of the usual diagnostic evaluation.
143 You admit a 68-year-old female with an acute stroke to the hospital. She has no other acute cardiovascular conditions. CT rules out a hemorrhagic event. You have determined that the patient is not a candidate for reperfusion therapy with alteplase or thrombectomy. You advise the nursing staff that you will be initiating antihypertensive therapy if the patient's blood pressure rises above a threshold of A) 120/80 mm Hg B) 140/90 mm Hg C) 160/100 mm Hg D) 180/110 mm Hg E) 220/120 mm Hg
E. Antihypertensive therapy should not be initiated in the first 48-72 hours for acute ischemic stroke patients unless certain criteria are met. The correct threshold for initiating therapy in this context is a blood pressure >220/120 mm Hg.
92 Question: A 67-year-old female who is a retired teacher presents with generalized itching. She believes she has acquired a skin infestation from small mites and provides a matchbox containing crusts, dried blood, and bits of skin. Which one of the following medications would be most likely to help this patient? A) Cholestyramine (Questran) B) Hydroxyzine (Vistaril) C) Ivermectin (Stromectol) D) Olanzapine (Zyprexa) E) Prednisone
Answer: D Explanation: This patient likely has delusions of infestation, and atypical antipsychotic medications such as olanzapine or risperidone can be effective. Cholestyramine is used for cholestatic jaundice, hydroxyzine for itching from urticaria, ivermectin for scabies, and prednisone for allergic reactions or inflammatory dermatitis.
142 The mother of a 6-month-old infant is concerned that her child's feet are "deformed." On examination the heel bisector line is between the third and fourth digits on the right foot and on the third digit on the left foot. You attempt to flex the feet, and both appear to be rigid. Which one of the following would you recommend as a corrective intervention? A) Night splints B) Adjustable orthotic shoes C) Braces D) Physical therapy E) Surgical correction
B. Adjustable orthotic shoes in infants not yet walking can be effective for treating metatarsus adductus, the condition described in the scenario.
139 A 44-year-old male with diabetes mellitus, hypertension, obesity, and chronic pain is on chronic opiate therapy. He comes to your office because of a lack of sex drive, decreasing strength, low overall energy levels, and hot flashes. After ruling out other causes, you confirm that he has a low total testosterone level on two separate early morning laboratory tests. He would like to start testosterone therapy. Which one of the following would be the most appropriate next step? A) Order a PSA level and perform a digital rectal examination B) Order LH and FSH levels C) Order chromosomal studies D) Discuss risks and benefits of testosterone replacement therapy and start low-dose replacement E) Inform him that testosterone replacement therapy would not be beneficial for him because of its high risk
B. After confirming low testosterone with two morning laboratory tests, the next step is to attempt to determine the cause of the low testosterone. Checking LH and FSH levels is recommended to evaluate for primary hypogonadism. If primary hypogonadism is present, chromosomal studies should be considered. Before initiating testosterone therapy, checking the patient's PSA level and performing a digital rectal examination are recommended, but in this case, the initial workup is not yet complete. It is crucial to discuss the risks and benefits of treatment, and as with all medications, it is recommended to start with the lowest dose needed. However, starting treatment in this case is premature. Evidence for testosterone replacement therapy is not as robust as desired, and it does carry risks, but as long as there are no contraindications, it can be initiated after a discussion of the risks and benefits.
136 A 6-week-old female is brought to your office by her parents to establish care after the family recently moved from out of state. The infant was born at term after an uncomplicated normal spontaneous vaginal delivery but failed her initial newborn hearing screen in the right ear only. Both parents are confident that she is able to hear out of both ears because she turns her head toward their voices regardless of where they are standing. A physical examination is within normal limits. Which one of the following would be the most appropriate next step in response to this patient's abnormal hearing screen? A) No further testing B) A bilateral audiology evaluation before 3 months of age C) A bilateral audiology evaluation at 6 months of age D) A bilateral audiology evaluation at 12 months of age E) A bilateral audiology evaluation immediately before entering kindergarten
B. All newborns should have a bilateral hearing screen completed before hospital discharge. For infants that fail the initial hearing screen in one or both ears, a repeat bilateral audiology evaluation should be performed before 3 months of age to ensure early identification of hearing loss and therefore maximize speech perception and development.
106. A 34-year-old male sees you because he was recently informed that a partner he had unprotected sex with last month has been diagnosed with HIV. You would advise this patient to initiate ongoing antiretroviral therapy A) immediately, because HIV testing is not necessary prior to initiation B) at the time of diagnosis of HIV infection C) when his CD4 cell count drops to <200 cells/ L D) when his CD4 cell count drops to <500 cells/ L E) when he develops an AIDS-defining illness
B. Antiretroviral therapy (ART) should be prescribed at the time of diagnosis of HIV infection unless the patient has expressed a desire to not initiate treatment. ART should not be delayed until the CD4 cell count drops to a predetermined level or until an AIDS-defining illness occurs. It is recommended to initiate prophylaxis for Pneumocystis pneumonia when the CD4 cell count drops below 200 cells/ L.
56 A 35-year-old male with type 1 diabetes asks you what his hemoglobin A1c goal should be. His blood pressure and lipids are well controlled, and he has not had any episodes of hypoglycemia. He recently saw an ophthalmologist and a podiatrist. You advise him that based on current American Diabetes Association recommendations his goal should be to keep his hemoglobin A1c below a threshold value of A) 6.0% B) 7.0% C) 8.0% D) 9.0%
B. Based on current American Diabetes Association recommendations, the goal for hemoglobin A1c in nonpregnant adults with diabetes is below 7.0%.
62 Chronic kidney disease is defined by abnormal kidney structure or function lasting a minimum of A) 2 months B) 3 months C) 6 months D) 12 months E) 24 months
B. Chronic kidney disease (CKD) is defined by abnormal kidney structure or function lasting greater than 3 months, with associated implications for health. Diagnostic criteria include a persistent glomerular filtration rate <60 mL/min/1.73 m2, albuminuria, urine sediment abnormalities, renal imaging abnormalities, and serum acid-base or electrolyte abnormalities.
53 An otherwise healthy 21-year-old male sees you for follow-up after a hospitalization for pneumonia. This was his second pneumonia infection of the year. He reports a history of multiple sinus infections and upper respiratory infections over the years that were treated with antibiotics on an outpatient basis. Laboratory studies reveal a normal CBC and a decreased IgA level. A trial of pneumococcal polysaccharide vaccine (PPSV23, Pneumovax 23) reveals no measurable response. This presentation is most consistent with A) selective IgA deficiency B) common variable immunodeficiency C) severe combined immunodeficiency D) DiGeorge syndrome E) Wiskott-Aldrich syndrome
B. Common variable immunodeficiency (CVID) is the most likely diagnosis in this case, as it often presents later in life with recurrent bacterial infections.
138 Which one of the following best explains the pathophysiology of cytokine storm? A) Anaphylaxis B) Immune dysregulation C) Immunodeficiency D) Normal physiologic response E) Serum sickness
B. Cytokine storm or cytokine release syndrome is caused by the release of cytokines and is characterized by fever, tachypnea, headache, tachycardia, hypotension, rash, and/or hypoxia. Cytokine storm can be triggered by certain therapies, pathogens, cancers, autoimmune conditions, and monogenic disorders. The normal inflammatory response involves recognition of a pathogen or injury, activation of a proportional response, and a return to homeostasis. However, cytokine storm involves immune dysregulation and immune-cell hyperactivation in which an overabundance of cytokines can cause collateral damage that may be worse than the benefit from the immune response itself. It is not considered a normal physiologic response, and it does not involve histamine release or anaphylaxis. Immune-cell hyperactivation rather than immunodeficiency is involved in cytokine storm. However, it is important to be aware of concurrent immunodeficiency since treatment for the immune hyperactivity can place patients at risk for secondary infections and illness. Serum sickness is associated with delayed hypersensitivity to foreign proteins from animal serums and is not involved in the pathophysiology of cytokine storm.
111 A 60-year-old male is referred to you by his dermatologist for additional workup of a symmetric vesicular rash that was diagnosed as dermatitis herpetiformis. This rash is associated with A) food allergy B) gluten sensitivity C) HIV infection D) immunodeficiency E) internal malignancy
B. Dermatitis herpetiformis is an immunologic response to ingested gluten and is pathognomonic for celiac disease. It may be the only presenting symptom of celiac disease, so there should be a high index of suspicion in patients presenting with a rash consistent with dermatitis herpetiformis. The papulovesicular rash is extremely pruritic and found on extensor surfaces such as elbows and knees, as well as the buttocks and scalp. It is more common in men than in women. The preferred initial diagnostic test includes a serum IgA transglutaminase-2 (TG2) antibody level, which has a 98% sensitivity and 98% specificity for the diagnosis of celiac disease. When celiac disease is strongly suspected despite a negative IgA TG2 antibody test, a total IgA level should be obtained. Dermatitis herpetiformis is not associated with food allergy, HIV, immunodeficiency, or internal malignancy.
30 A 40-year-old female with a history of breast cancer treated with mastectomy and adjuvant chemotherapy presents to the exocrine gland dysfunction with increased tear production. Which one of the following is the most likely explanation for this patient's symptoms? A) Blockage of lacrimal glandens duct by tumor recurrence C) Nasolacrimal duct obstruction D) Loss of corneal sensation E) Chronic irritation
B. Exocrine gland dysfunction (eg, dry eyes, dry mouth) is a common complication of radiation therapy to the head and neck. The lacrimal and salivary glands are particularly susceptible to radiation-induced damage. Dry eyes result from destruction of the lacrimal gland and the conjunctival goblet cells. Other complications include cataracts and retinopathy.
12 A 33-year-old gravida 2 para 1 presents to the hospital at 35 weeks estimated gestation with premature rupture of membranes. A decision is made to manage the pregnancy expectantly and delay delivery unless signs of infection or fetal distress are noted. Based on current evidence, expectant management rather than immediate delivery increases the risk of which one of the following complications? A) Cesarean delivery B) Antepartum or postpartum maternal hemorrhage C) Time spent in the neonatal intensive-care unit D) Neonatal sepsis E) Perinatal or infant mortality
B. Expectant management appears to be associated with better neonatal outcomes, decreasing the risk of cesarean delivery, neonatal respiratory distress, mechanical ventilation, time spent in the neonatal intensive-care unit, and time spent in the hospital. It does increase the risk of maternal antepartum or postpartum hemorrhage and intrapartum fever, but not neonatal sepsis or perinatal or infant mortality.
149 A 4-year-old male is brought to your office by his mother because of a 2-day history of watery diarrhea and vomiting, and you diagnose acute gastroenteritis. On examination, his mucous membranes are sticky, and he has decreased tear production, but his overall appearance is normal and his eyes are not sunken. Using the Clinical Dehydration Scale, you estimate that he has mild (3%-6%) dehydration. Which one of the following should you recommend? A) Water as tolerated B) Half-strength apple juice followed by preferred fluids C) The bananas, rice, applesauce, and toast (BRAT) diet D) Intravenous fluids E) Metoclopramide (Reglan)
B. For mild dehydration in acute gastroenteritis, the recommendation is water as tolerated or half-strength apple juice followed by preferred fluids.
131 A 72-year-old female presents with pain, swelling, and decreased range of motion in her right great toe for several months. There is no history of injury or overuse. On examination, the metatarsophalangeal joint is swollen and mildly tender, but not red. Dorsiflexion and plantar flexion are approximately 30°. A radiograph shows joint space narrowing and a small bone spur. Recommended management at this time would be A) stretching and strengthening exercises B) a rigid shoe insert C) ibuprofen D) a corticosteroid injection E) surgical referral
B. Hallux rigidus affects as many as 50% of women and 40% of men by the age of 70. It is usually due to osteoarthritis of the metatarsophalangeal (MTP) joint and presents as decreased range of motion, swelling, and pain. With progression of the condition, flare-ups become more frequent and more severe, and it can be mistaken for gout. Initial treatment is restriction of motion across the MTP joint. A stiffening shoe insert does relieve pain, and most patients see improvement without surgery. Custom orthotics, rigid inserts, or hard-soled shoes are options that are more effective than NSAIDs. Corticosteroid injections, preferably administered with ultrasound guidance, and surgery are reserved for those who fail to respond to more conservative measures. Stretching and strengthening exercises are recommended for plantar fasciitis more so than for hallux rigidus.
121 A 5-year-old female is brought to your office by her mother because she has not wanted to eat or drink anything since she woke up this morning. She has also had a temperature of 100.2°F for the last 2 days. She usually attends day care 3 days a week but has been kept at home because of the fever. She is up to date on all age-appropriate recommended immunizations. On examination you notice several oral ulcerations that are painful to palpation, and several erythematous, vesicular lesions on the patient's palms. Which one of the following is the most likely diagnosis? A) Erythema multiforme B) Hand-foot-and-mouth disease C) Herpetic gingivostomatitis D) Oral candidiasis E) Varicella
B. Hand-foot-and-mouth disease is a viral illness caused by human enteroviruses and coxsackie viruses that presents in the spring to the fall, generally in children <10 years of age. It is characterized by a low-grade fever, uncomfortable oral lesions, and a papular to vesicular rash on the hands and soles of the feet. Hydration and pain control with acetaminophen or ibuprofen are the mainstays of treatment. Erythema multiforme is characterized by target lesions on the trunk, face, and limbs, as well as vesicular lesions that can affect oral, genital, and ocular mucosa. It is most common in young adults 20-40 years of age and is slightly more predominant in males. Herpetic gingivostomatitis can be associated with fever, decreased appetite, and oral vesicles that can be found on the lips, palate, tongue, and gums, but not on the palms of the hands or soles of the feet. Oral candidiasis is characterized by a white film that could be scraped off and generally is not associated with a fever. Varicella causes a very pruritic vesicular rash that starts on the face and trunk and then spreads to the remainder of the body. Children are routinely vaccinated against varicella, making it an unlikely diagnosis in this patient who is up to date on all age-appropriate recommended immunizations.
157 A 1-month-old infant is brought to your office by her parents for routine follow-up. The infant was born at 35 weeks and 5 days gestation by normal vaginal delivery after induction of labor for maternal preeclampsia with severe features. The infant was discharged with the mother on the third day of life. There were no additional complications noted. Today the parents report exclusive breastfeeding and tell you the infant spits up after longer feedings. Voiding and stooling are as expected, and the infant's development is normal. The weight at birth was at the 20th percentile, and today's weight is at the 25th percentile. Which one of the following would be the most appropriate intervention at this time? A) Caloric supplementation in addition to breastfeeding B) Iron supplementation C) Vitamin E supplementation D) Referral for early intervention
B. In 2018, 1 in 10 infants born in the United States were preterm, with significant racial and ethnic differences noted. Breastfed infants born before 37 weeks gestation should receive iron supplementation at 2 mg/kg/day after 1 month of life. This infant does have some physiologic reflux but since this infant appears asymptomatic, the parents should be counseled on behavioral techniques to reduce spitting up, as there is no clear long-term benefit to antireflux medication. This infant's growth and development are normal so there is no indication for caloric fortification of breast milk, which is more appropriate for small-for-gestational-age infants or those born below the 10th percentile. There is no specific recommendation for micronutrient supplementation other than iron and vitamin D, so there is no indication to initiate vitamin E supplementation. This child should be screened for developmental delay at each office visit, but there is currently no evidence of delay so referral to early intervention is not indicated.
3 A 68-year-old female presents with a 2-month history of watery diarrhea. She has not had any blood or pus in her stools, and the stools are not oily. She has not had any history of fever, chills, or weight loss, and has not traveled recently. She smokes one pack of cigarettes per day. Her medications include ibuprofen, sertraline (Zoloft), and pantoprazole (Protonix). A CBC, metabolic panel, C-reactive protein level, IgA anti-tissue transglutaminase level, total IgA level, and stool guaiac test are all normal. Which one of the following tests would be most likely to yield a diagnosis? A) Clostridioides (Clostridium) difficile toxin B) Colonoscopy C) Fecal calprotectin D) A stool culture E) Stool examination for ova and parasites
B. In patients with chronic nonbloody diarrhea, the differential diagnosis includes microscopic (lymphocytic or collagenous) colitis. The mucosa appears normal on colonoscopy but a biopsy will show lymphocytic infiltration of the epithelium. The etiology of this is unknown but there are several risk factors to consider, including older age, female sex, and smoking status. Drugs with a high level of evidence for causing microscopic colitis include NSAIDs, proton pump inhibitors, sertraline, acarbose, aspirin, and ticlopidine. Clostridioides (Clostridium) difficile should be suspected in individuals who have taken antibiotics in the past 3 months. Fecal calprotectin is elevated in inflammatory diarrhea such as Crohn's disease or ulcerative colitis. A stool culture would be indicated if there is suspicion of an infectious bacterial diarrhea such as Shigella or Salmonella, but these bacteria tend to cause bloody diarrhea. Checking for a parasitic infection should be considered for patients with a history of recent travel or exposure to unpurified water. Ref: Burgers K, Lindberg B, Bevis ZJ: Chronic diarrhea in adults: Evaluation and differential diagnosis. Am Fam Physician 2020;101(8):472-480.
29 A 58-year-old male is brought to your office by his wife for follow-up after an urgent care clinic visit 3 days ago where he was diagnosed with pneumonia based on a chest radiograph. He has been taking his antibiotics as prescribed. He has a blood pressure of 110/70 mm Hg, a respiratory rate of 27/min, and a temperature of 38.7°C (101.7°F). Which one of the following would be the most appropriate next step? A) Chest radiography B) Pulse oximetry C) CBC D) Observation E) Reassurance
B. In patients with community-acquired pneumonia who do not respond to initial empiric antibiotic therapy, diagnostic testing (eg, sputum or blood culture, Legionella and pneumococcal urinary antigen tests) should be performed to guide further management. These tests are most useful when performed prior to the initiation of antibiotics. However, in this patient who has already started treatment, testing may yield false-negative results.
145 Which one of the following is an indication for a radionuclide thyroid uptake scan for a patient with a single thyroid nodule confirmed by ultrasonography? A) The presence of antithyroid antibodies B) A low TSH level C) An elevated TSH level D) A normal TSH level and a tender nodule E) The presence of the thyroid nodule regardless of other findings
B. In thyroid nodule evaluation, if TSH is low, a radionuclide scan is indicated. It helps differentiate hyperfunctioning nodules.
9 You are providing end-of-life care for a 53-year-old female with end-stage colon cancer. Her family reports that she is having significant abdominal pain, nausea, and vomiting, and she is not able to tolerate oral intake. You suspect a malignant bowel obstruction. Which one of the following interventions would be most likely to significantly improve her symptoms? A) Medical cannabis B) Dexamethasone C) Morphine D) Octreotide (Sandostatin) E) Polyethylene glycol (MiraLAX)
B. Malignant bowel obstruction is a common issue with gastrointestinal cancers. Corticosteroids can help alleviate these symptoms, which is the focus in end-of-life care. Corticosteroids have numerous beneficial effects in these situations, such as central antiemetic, anti-inflammatory, antisecretory, and analgesic effects. Intravenous dexamethasone is generally recommended at a dosage of 4 mg 3-4 times daily for malignant bowel obstruction because it has much greater anti-inflammatory effect than methylprednisolone. Although octreotide is commonly used for this purpose, there is little evidence to support its use. Medical cannabis can be used to treat nausea and vomiting in end-of-life care but is not effective for bowel obstruction. Morphine can be used to treat pain and end-of-life dyspnea, but not nausea and vomiting. The use of polyethylene glycol for a malignant obstruction could worsen the patient's symptoms significantly. Ref: Feuer DJ, Broadley KE: Corticosteroids for the resolution of malignant bowel obstruction in advanced gynaecological and gastrointestinal cancer. Cochrane Database Syst Rev 2000;2000(2):CD001219. 2) Patel A, Garg R: Role of steroids in malignant bowel obstruction. Palliat Med Hosp Care Open J 2016;2(2):30-36. 3) Albert RH: End-of-life care: Managing common symptoms. Am Fam Physician 2017;95(6):356-361.
11 A 60-year-old male with diabetes mellitus and hypertension sees you for routine follow-up. He has no acute health concerns during today's visit. His current medications include metformin (Glucophage), lisinopril (Prinivil, Zestril), and hydrochlorothiazide. He smokes cigarettes and has a 40-pack-year smoking history. His vital signs and a physical examination are normal. An in-office dipstick urinalysis reveals 1+ blood and trace protein but is otherwise normal. Which one of the following would be the most appropriate follow-up? A) Repeat dipstick urinalysis in 3 months B) Microscopic urinalysis C) Renal ultrasonography D) CT urography E) Referral for cystoscopy
B. Microscopic hematuria, also known as microhematuria, is defined as 3 RBCs/hpf on microscopy. Dipstick analysis alone is insufficient to diagnose microscopic hematuria. Confirmation with microscopic analysis is needed. The current guideline recommends further evaluation based on the patient's overall risk of genitourinary malignancy. For this patient, the next step would be microscopic urinalysis to determine the presence and quantify hematuria. If confirmed, CT urography and cystoscopy may be indicated due to his high-risk history. Repeating the dipstick analysis in 3 months is inappropriate in this situation.
141 A patient's office spirometry results demonstrate an obstructive pattern. This would be seen with which one of the following? A) Asbestosis exposure B) Cystic fibrosis C) Idiopathic pulmonary fibrosis D) Nitrofurantoin exposure E) Sarcoidosis
B. Office spirometry can help narrow down a differential diagnosis. An obstructive pattern is seen in conditions like cystic fibrosis. Other causes of an obstructive pattern include asthma, COPD, 1-antitrypsin deficiency, and bronchiectasis, among others.
66 A 70-year-old male presents with a 2-year history of gradually progressive exertional dyspnea associated with a dry cough and fatigue. A physical examination reveals bilateral basilar fine inspiratory crackles on lung auscultation and acrocyanosis. A chest radiograph demonstrates hazy opacities and reticular infiltrates of both lower lung fields. You suspect interstitial lung disease. Assuming that no underlying connective tissue disease is identified on serologic testing, which one of the following additional studies could confirm a diagnosis of idiopathic pulmonary fibrosis for this patient, potentially preventing the need for a subsequent lung biopsy? A) Spirometry B) High-resolution chest CT C) Polysomnography D) Echocardiography E) Right heart catheterization
B. Once ILD is suspected, further evaluation is indicated to determine a more specific diagnosis. Of the options listed, only high-resolution chest CT has the potential to provide a specific diagnosis of IPF, which usually has a characteristic pattern of bilateral reticulation and honeycombing in the lung periphery and in the lower lobes termed usual interstitial pneumonia.
44 Polypharmacy increases the risk of adverse health outcomes. According to the Choosing Wisely campaign, adding to a threshold of how many medications in a patient's regimen should prompt a thorough review to determine if any of the medications can be discontinued? A) 3 B) 5 C) 7 D) 10
B. Polypharmacy, defined as regular use of five or more medications, increases the risk of adverse medical outcomes. According to the Choosing Wisely campaign, any prescriptions beyond a threshold of five medications should trigger a thorough review of the patient's complete regimen, including over-the-counter medications and dietary supplements, to determine if any of the medications can be discontinued.
127 An asymptomatic 53-year-old female recently underwent a right breast lumpectomy and radiation therapy with curative intent. Over the next 5 years routine surveillance should include A) annual unilateral left breast mammography B) annual bilateral mammography C) annual bilateral mammography and radionuclide bone scans D) biannual bilateral mammography E) biannual bilateral mammography and annual radionuclide bone scans
B. Primary care physicians should ensure that their patients who have undergone treatment for breast cancer follow the recommendations of their oncologist, as well as receive a history evaluation and health maintenance examination every 3-6 months for 3 years, every 6-12 months for 2 more years, and then on an annual basis. For ongoing surveillance only annual mammography is recommended (SOR A), which is bilateral in breast-conserving therapy and unilateral following a mastectomy. Other surveillance testing such as radionuclide bone scans, PET scans, and biomarkers should not be performed in asymptomatic patients who received curative treatment.
160 A 66-year-old male presents to your office with a 1-week history of dyspnea with minimal exertion but no chest pain. He has had mild hemoptysis. An examination reveals a pulse rate of 100 beats/min but no other remarkable findings. A chest radiograph, CBC, and metabolic panel are normal, but his D-dimer level is elevated at 750 ng/mL (N <500). Which one of the following would be the next step in the evaluation? A) A BNP level B) CT pulmonary angiography C) An EKG D) Pulmonary arteriography E) A ventilation-perfusion scan
B. Pulmonary embolus is reliably diagnosed with CT pulmonary angiography (CTA), but there is now a simple diagnostic algorithm to reduce the reliance on CTA. The simplified recommendations for ordering CTA are a D-dimer ≥1000 ng/mL, or a D-dimer that is >500 ng/mL and hemoptysis, signs of deep vein thrombosis, or a suspicion that pulmonary embolism is the most likely diagnosis. A BNP level would be useful in detecting heart failure, and an EKG would be more helpful if ischemic heart disease were suspected. Pulmonary arteriography is invasive and carries a higher risk. A ventilation-perfusion scan has less risk but is not as accurate.
76 A 30-year-old female who is an established patient calls your office to request a test for COVID-19. The patient spent several hours inside the home of another individual who just received a positive COVID-19 test result. She states that her sense of taste seems diminished, but she has no respiratory symptoms and otherwise feels well. Which one of the following is the typical incubation period for COVID-19? A) 1 day B) 5 days C) 14 days D) 30 days
B. SARS-CoV-2 is a respiratory coronavirus that is responsible for COVID-19. Knowledge of the natural history of the viral infection will inform testing strategies and many other aspects of counseling of patients. The incubation period measures the time from exposure to symptom onset. The typical incubation period for COVID-19 is approximately 4-5 days, though it can range from 1-14 days.
31 A 62-year-old male was hospitalized for an upper gastrointestinal (GI) bleeding episode and the gastroenterologist arranges for you to provide follow-up care. The patient's medications include atorvastatin (Lipitor), metformin (Glucophage), lisinopril/hydrochlorothiazide (Zestoretic), citalopram (Celexa), and omeprazole (Prilosec). Which one of the following medications in his current regimen increases the risk of an upper GI bleeding episode? A) Atorvastatin B) Citalopram C) Lisinopril/hydrochlorothiazide D) Metformin E) Omeprazole
B. SSRIs such as citalopram increase the risk of upper gastrointestinal (GI) bleeding by 55%, according to meta-analysis studies. Omeprazole, a proton pump inhibitor, decreases the risk of recurrent GI bleeding.
119 A 60-year-old female sees you for recent syncopal episode. She tells you that she recently fainted during a routine walk with a friend, who told her that everything seemed fine and suddenly the patient fell to the ground and was unresponsive. The patient denies any symptoms. She did not have any associated symptoms, abnormal movement, or incontinence and was not injured in the fall. The episode lasted a few seconds and then she regained consciousness, stood, and finished the walk without difficulty. She has not had any similar episodes before this. Her PMH and FH are unremarkable. She's otherwise heakthy and takes no medications except. An examination and EKG is normal. Which one of the following would most likely help confirm the diagnosis? A) Orthostatic blood pressure measurements B) An external cardiac event monitor C) Ultrasonography of the carotid arteries D) CT of the head E) MRI of the brain
B. Syncope is generally classified into three broad categories: cardiac, neurally mediated, and orthostatic hypotension. The history of an abrupt unprovoked episode of syncope in this patient suggests a cardiac source. The most frequent cause of cardiac-associated syncope is an arrhythmia. A clinical history suggestive of arrhythmia places the patient in a higher risk stratification. An EKG should be performed. Continuous cardiac monitoring correlates symptoms with cardiac rhythm. Options for monitoring include a Holter monitor for up to 72 hours, an external cardiac event monitor or loop recorder for up to 4-6 weeks, and an implantable loop recorder for up to 3 years.
60 The most common symptom of obstructive sleep apnea is A) cough B) excessive sleepiness C) leg swelling D) palpitations E) weight gain
B. The most common symptom of obstructive sleep apnea is excessive sleepiness. Cough, leg swelling, palpitations, and weight gain are not among the most common symptoms of OSA.
84 A 34-year-old female with asthma sees you for routine follow-up. She tells you that she uses her short-acting -agonist (SABA) approximately twice a week. Which one of the following management strategies would you recommend for prevention of exacerbations? A) Continued use of a SABA as needed B) An inhaled corticosteroid (ICS)/long-acting -agonist (LABA) as needed C) A daily maintenance ICS/LABA D) A daily maintenance ICS plus a SABA as needed E) A daily maintenance ICS plus a daily leukotriene receptor antagonist
B. The recommended management strategy for preventing exacerbations is: An inhaled corticosteroid (ICS)/long-acting -agonist (LABA) as needed - Recent evidence supports the use of ICS/LABA as needed for mild asthma, which is as effective as a daily maintenance ICS plus a short-acting -agonist (SABA).
125 A 15yo M presents to the ED after falling on his bicycle and hitting his head. He's brought by his friend's father. The patient tells you that he has a headache and slight nausea but otherwise feels fine. The triage nurse is unable to reach a parent or guardian. While continuing to try to reach the child's parent or guardian, which one of the following would be most appropriate regarding care for this patient? A) Observe him in the waiting room until his parent or guardian can be reached B) Complete the initial screening and provide any emergent treatment C) Ask two physicians to document an intent to treat prior to any screening or emergent treatment D) Contact a court official to obtain an emergency consent for medical treatment prior to any screening or emergent treatment E) Contact the child protective services agency and arrange for emergency guardianship prior to any screening or emergent treatment
B. There are many circumstances in which a minor may present to an emergency department (ED) for evaluation and treatment without a parent or guardian. Federal law requires that the ED medical providers complete the initial medical screening and evaluation of minors even if parental consent cannot be obtained. In addition, treatment of emergencies is required, even without parental consent. This should all be done while working diligently to obtain consent from a parent, family member, or legal guardian. The medical providers should also explain everything to the minor in terms that the patient can understand.
48 A 70-year-old female with a history of coronary artery disease, a femorofemoral bypass 3 years ago, and hypertension sees you for a follow-up visit. She has intermittent right arm pain that is worse with exercise. The pain increases with all arm exercises and improves with rest. The patient's blood pressure is 140/70 mm Hg in the left arm and 120/64 mm Hg in the right arm. Which one of the following would be the most appropriate next step? A) Radiographs of the right shoulder B) Arterial duplex ultrasonography of the upper extremities C) MR angiography D) No imaging, and referral to physical therapy
B. This patient has peripheral artery disease (PAD) of the right arm. PAD of the upper extremities is characterized by pain with exertion and can cause gangrene and ulceration. It is more common in patients who have had lower extremity occlusive disease. A blood pressure differential of 15 mm Hg between arms suggests stenosis and warrants further testing. Initial testing in symptomatic patients includes arterial duplex ultrasonography of the upper extremities. CT angiography and MR angiography may be appropriate to clarify the diagnosis or plan intervention. Neither radiography nor physical therapy would be appropriate.
2 A 28-year-old female presents for evaluation of nasal congestion, sneezing, watery eyes, and postnasal drip. This has been an intermittent issue for her every spring and she would like to manage it more effectively. Which one of the following treatments has been shown to be the most effective and best tolerated first-line therapy for this patient's condition? A) A leukotriene receptor antagonist B) Intranasal corticosteroid monotherapy C) Intranasal corticosteroids plus an oral antihistamine D) Inhaled corticosteroids E) Annual triamcinolone injections
B. This patient has seasonal allergic rhinitis. A joint guideline statement from the American Academy of Allergy, Asthma, and Immunology/American College of Allergy, Asthma, and Immunology Joint Task Force on Practice Parameters recommends that monotherapy with intranasal corticosteroids should be prescribed initially in patients 12 years of age rather than combined treatment with oral antihistamines because data has not shown an additional benefit to adding the antihistamine. Higher patient adherence and tolerance and fewer side effects were seen with the monotherapy regimen. High-quality evidence indicates that intranasal corticosteroids were more effective than leukotriene receptor antagonists. Inhaled corticosteroids and triamcinolone injections are not appropriate first-line options for the treatment of seasonal allergic rhinitis. Ref: Hauk L: Treatment of seasonal allergic rhinitis: A guideline from the AAAAI/ACAAI Joint Task Force on Practice Parameters. Am Fam Physician 2018;97(11):756-757.
54 Which one of the following vitamins is a well-established therapy for the treatment of nausea and vomiting in pregnancy? A) Vitamin A B) Vitamin B6 C) Vitamin B12 D) Vitamin C E) Vitamin E
B. Vitamin B6 (pyridoxine) is a well-established therapy for pregnancy-related nausea and vomiting.
39 A 52-year-old male with known hypertension and hyperlipidemia comes to your office for a follow-up visit. His last visit was more than a year ago. He was unemployed for several months and lost his health insurance. Two months ago he ran out of his medications, which included amlodipine (Norvasc), hydrochlorothiazide, and atorvastatin (Lipitor). Which one of the following would be the most appropriate next step? A) Administer clonidine (Catapres), 0.1 mg orally, and recheck his blood pressure in 30 minutes B) Administer nifedipine (Procardia), 60 mg orally, and recheck his blood pressure in 30 minutes C) Order laboratory studies to look for end-organ damage, and tell him to restart his previous medications D) Admit him to the intensive-care unit for intravenous treatment to lower his blood pressure E) Call 911 and have him transported to the emergency department
C. A hypertensive urgency is defined as a confirmed blood pressure >180/110-120 mm Hg without symptoms or signs of end-organ damage. Patients without symptoms in the setting of severe hypertension rarely have end-organ damage. The most common cause of hypertensive urgency in patients with known hypertension is nonadherence to the use of antihypertensive medications.
134 A 64-year-old female presents to the emergency department with a 10-day history of increasing shortness of breath and mild tachycardia. On examination, she has an oxygen saturation of 75% on room air. Which one of the following additional findings would suggest a diagnosis of acute respiratory distress syndrome (ARDS)? A) Improved oxygen saturation with supplemental oxygen B) Improvement of her symptoms with diuretic therapy C) Bilateral airspace opacities seen on a chest radiograph D) A flattened diaphragm seen on a chest radiograph E) A right lower lobe infiltrate seen on a chest radiograph
C. Acute respiratory distress syndrome (ARDS) will often present similarly to pneumonia and heart failure with dyspnea, hypoxemia, and tachypnea. ARDS typically does not respond to supplemental oxygen or diuretic therapy. Patients decompensate quickly and usually require mechanical ventilation. Chest radiographic findings include bilateral airspace opacities but not a localized infiltrate as with pneumonia, venous congestion or cardiac enlargement as with heart failure, or a flattened diaphragm (associated with COPD).
118 A 45-year-old male comes to your office for a routine health maintenance examination. He reports frequently drinking five 12-oz beers per day despite trying to cut down. He tells you that he had an abnormal liver profile in the past and sometimes drives under the influence of alcohol. He asks for your help to decrease his alcohol consumption. Which one of the following medications would be most likely to help this patient decrease his alcohol use? A) Baclofen intrathecal (Lioresal) B) Disulfiram (Antabuse) C) Naltrexone (Vivitrol) D) Pregabalin (Lyrica) E) Sertraline (Zoloft)
C. Alcohol use causes preventable morbidity and mortality in the United States. Alcohol use disorder (AUD) affects approximately 9% of men in the United States. Family physicians should be familiar with evidence-based interventions to treat this disorder. Naltrexone has been shown to decrease heavy drinking, daily drinking, and the amount of alcohol consumed (SOR A). A 2018 Cochrane review did not support the use of baclofen for AUD. Evidence does not support the use of disulfiram for AUD but it may be offered in selected circumstances, particularly when patients do not tolerate other options and their goal is abstinence. There is limited evidence on the use of pregabalin for AUD but the available data does not support its use. Antidepressants are not effective for AUD unless these is a coexisting mental health disorder such as depression.
103 While on rounds in the newborn nursery, you receive a call about a 2-day-old infant born at 39 weeks gestation. According to the American Academy of Pediatrics standard treatment guidelines for infants at high risk of bilirubin encephalopathy, the infant has an elevated total serum bilirubin level that is approaching the threshold for initiating phototherapy. Which one of the following additional factors would be the strongest indication for phototherapy in this infant? A) East Asian ethnicity B) Exclusive breastfeeding C) A positive direct antibody titer (Coombs test) D) A sibling with a history of neonatal jaundice E) Untreated maternal group B streptococcal colonization
C. Although up to 84% of term newborns experience neonatal jaundice, severe hyperbilirubinemia (total serum bilirubin level >20 mg/dL) occurs in <2% of term infants. Prompt identification and management of severe hyperbilirubinemia is critical due to the risk of neurologic injury from untreated bilirubin toxicity. Risk factors for toxicity include earlier gestational age at birth, hemolysis, sepsis, acidosis, G6PD deficiency, lethargy, asphyxia, temperature instability, acidosis, and hypoalbuminemia. Hemolysis, associated with a positive direct antibody titer (Coombs test), is the most significant risk factor for developing acute bilirubin encephalopathy and impacts the treatment threshold for initiation of phototherapy. East Asian ethnicity, exclusive breastfeeding, a sibling with a history of neonatal jaundice, and untreated maternal group B streptococcal colonization are risk factors for severe hyperbilirubinemia but do not directly impact the phototherapy treatment threshold.
74 A 3-year-old male undergoes surgery for intestinal malrotation (volvulus). A preoperative comprehensive metabolic panel was normal. The patient's postoperative potassium level is 4.6 mEq/L (N 3.4-4.7). In addition to maintenance potassium, which one of the following fluids should be administered in the postoperative period? A) 0.2% NaCl with 5% dextrose B) 0.45% NaCl with 5% dextrose C) 0.9% NaCl with 5% dextrose D) 3% saline E) 5% dextrose in water
C. Based on well-designed randomized, controlled trials and systematic reviews, the American Academy of Pediatrics recommends the use of isotonic solutions with adequate potassium chloride and dextrose for maintenance intravenous fluids in children. This approach significantly reduces the risk of hyponatremia without increasing other risks such as hypernatremia and acidosis. Hypotonic commercial solutions such as 0.2% sodium chloride and 0.45% sodium chloride do not contain the appropriate sodium concentration, and 3% saline and 5% dextrose in water would not be appropriate for maintenance intravenous fluids in children.
67 A 47-year-old female presents with a 2-month history of generalized left shoulder pain. She does not have any ongoing illnesses. On examination you note generally impaired range of motion in the shoulder, but a radiograph of the shoulder is normal. You diagnose adhesive capsulitis. Which one of the following tests would be most appropriate at this time? A) Antinuclear antibody B) Erythrocyte sedimentation rate C) Hemoglobin A1c D) IgA tissue transglutaminase E) MRI without contrast
C. Both diabetes mellitus and hypothyroidism have been found to have a prevalence of 25%-50% in patients with adhesive capsulitis. Consideration should be given to testing for both of these conditions when making the diagnosis of adhesive capsulitis. An erythrocyte sedimentation rate is most appropriate at this time.
4 A 23-year-old male with opioid use disorder requests buprenorphine therapy. He is still actively using immediate-release oxycodone (Roxicodone) and he took a dose 2 hours ago. This patient should begin buprenorphine induction A) now B) in 2 hours C) 8-12 hours after his last opioid use D) 24 hours after his last opioid use E) 1 week after his last opioid use
C. Buprenorphine is a partial opioid agonist. In order to reduce the risk of precipitated withdrawal, buprenorphine induction should begin once the patient is exhibiting signs of mild to moderate withdrawal, usually 8-12 hours after the last opioid use. Waiting until a patient goes through full withdrawal increases the chances that the patient will revert back to using opioids. Ref: Zoorob R, Kowalchuk A, Mejia de Grubb M: Buprenorphine therapy for opioid use disorder. Am Fam Physician 2018;97(5):313-320.
117 A 68-year-old male with a history of COPD, htn, and hld presents with a worsening cough and DOE over the past 3m. His symptoms were previously well controlled with tiotropium (Spiriva) daily and albuterol (Proventil, Ventolin) as needed, and he has not had any COPD exacerbations in the past year. He has no change in sputum. Recently he has been using his albuterol inhaler several times a day to help relieve his SOB w/DOE. PE showed only a BP of 144/82 mm Hg and a mild prolonged expiratory phase. According to GOLD, which one of the following would be the most appropriate next step in the management of this patient's symptoms? A) Add azithromycin (Zithromax) B) Add inhaled fluticasone (Flovent) C) Add inhaled salmeterol (Serevent) D) Add inhaled fluticasone/salmeterol (Advair) E) Discontinue tiotropium and start inhaled fluticasone
C. COPD is currently the third leading cause of death in the United States and is commonly treated by primary care providers. In patients on monotherapy with a long-acting bronchodilator such as a long-acting muscarinic agonist (LAMA) or long-acting -agonist (LABA) who have continued dyspnea, the Global Initiative for Chronic Obstructive Lung Disease (GOLD) guidelines recommend escalating therapy to two bronchodilators. This patient has persistent dyspnea and is being treated with a single agent, a LAMA, so his regimen needs to be escalated to include a LABA such as salmeterol. Once the symptoms are stabilized, treatment can be de-escalated to a single agent. For patients with frequent COPD exacerbations or with a diagnosis of asthma and COPD, the guidelines recommend adding an inhaled corticosteroid (ICS) such as fluticasone to a LABA, LAMA, or both.
126 A 32-year-old male comes to your office for a follow-up visit after spontaneously passing a renal stone. This is his third such episode. Analysis of a previous stone showed that it was composed of calcium oxalate. After the first episode he increased his fluid intake and is drinking more than 3 L of water daily. He does not take any medications. An examination is unremarkable. Which one of the following should you recommend at this time? A) A very-low-calcium diet B) Furosemide (Lasix) C) Potassium citrate D) Probenecid
C. Calcium stones, composed of either calcium oxalate and/or phosphate, account for up to 90% of all stones in adults in developed countries. Increasing fluid intake to 2.5-3 L/day is the most important lifestyle modification to prevent recurrent kidney stones. A diet rich in fiber and vegetables with normal calcium content (1-1.2 g/day), limited sodium intake (4-5 g/day), and limited animal protein intake (0.8-1 g/kg/day) is strongly encouraged. Reduction of BMI by dietary modification and increased exercise is also recommended. Citrate supplementation with potassium citrate is recommended for preventing calcium stones that recur despite lifestyle modifications. Thiazide diuretics in higher dosages, such as 50 mg daily of hydrochlorothiazide, have also been shown to be effective in preventing calcium stone formation. Allopurinol is also an effective option. Elimination of all calcium from the diet, such as a low- to very-low-calcium diet, is discouraged as it not only increases stone formation but may also result in bone demineralization. Furosemide increases urinary calcium excretion and would increase the likelihood of calcium stone formation. Probenecid is not recommended in patients with uric acid stones, as it is a uricosuric agent.
101 A 71-year-old female with a history of well-controlled hypertension, diabetes mellitus, and osteoporosis presents with a 2-day history of fever, chills, and a productive cough. She lives at home with her husband, who has not noted any confusion but says she has been weak and unable to bathe herself. On examination, the patient has a temperature of 38.2°C (100.8°F), a blood pressure of 110/68 mm Hg, unlabored respirations at a rate of 22/min, and an oxygen saturation of 94% on room air. Posteroanterior and lateral chest radiographs show an infiltrate in the right middle lobe. Which one of the following would be the most appropriate treatment for this patient? A) Azithromycin (Zithromax) B) Amoxicillin plus metronidazole (Flagyl) C) Amoxicillin/clavulanate (Augmentin) plus azithromycin D) Azithromycin plus levofloxacin E) Clindamycin (Cleocin) plus doxycycline
C. Community-acquired pneumonia (CAP) is an infection of the lung parenchyma. This patient has CAP in the presence of a significant comorbidity (diabetes mellitus). After CAP is diagnosed, the first decision is whether hospitalization is needed. In this case, the patient can be treated as an outpatient. For outpatients with comorbidities, amoxicillin/clavulanate is a possible treatment option, paired with a macrolide. Macrolides like azithromycin are the treatment of choice for previously healthy outpatients with no history of antibiotic use within the past 3 months. Azithromycin monotherapy, amoxicillin plus metronidazole, azithromycin plus levofloxacin, or clindamycin plus doxycycline would not be appropriate for this patient with a significant comorbidity.
59 A 74-year-old male with type 2 diabetes, severe peripheral artery disease, and a history of tobacco use is admitted to the hospital with wet gangrene of his right foot that does not improve with appropriate wound care and antibiotics. His vascular surgeon recommends a below-the-knee amputation of the right leg but the patient has not given his consent. Which one of the following would suggest that this patient lacks capacity to make this decision? A) He has a fear of having surgery B) He wants to consult with his family first C) He consistently believes that his foot will improve D) He understands that he may die if he forgoes the amputation E) He states that his brother had the same surgery and died a week later
C. Consistently believing that his foot will improve despite evidence to the contrary suggests impaired capacity to make a medical decision.
156 You diagnose nonvalvular atrial fibrillation in a 54-year-old male. His CHA2DS2-VASc score is 2. Which one of the following should you recommend as first-line therapy for stroke prevention? A) No antithrombotic therapy B) Aspirin C) A direct oral anticoagulant D) Low molecular weight heparin E) A vitamin K antagonist
C. Direct oral anticoagulants such as apixaban, betrixaban, dabigatran, edoxaban, and rivaroxaban are first-line agents for prevention of stroke in patients with nonvalvular atrial fibrillation with a CHA2DS2-VASc score ≥2 in men or ≥3 in women. For patients with atrial fibrillation without valvular heart disease, forgoing antithrombotic therapy is only appropriate in patients with a CHA2DS2-VASc score of 0 in men and 1 in women. Aspirin should not be considered a substitute for anticoagulation but may be suggested for patients with an unprovoked deep vein thrombosis or pulmonary embolism who do not wish to receive lifelong anticoagulation. Low molecular weight heparin is recommended as the anticoagulant of choice in patients with cancer and venous thromboembolism, although direct anticoagulants may be appropriate in some situations. If a patient has moderate to severe mitral stenosis or a mechanical valve, then vitamin K antagonists are the preferred agent.
57 You diagnose major depressive disorder in a 69-year-old male. He has benign prostatic hyperplasia and is treated with prazosin (Minipress) for nocturia but he is otherwise healthy. He agrees to cognitive-behavioral therapy and starting an antidepressant but is concerned about side effects, especially falls. Which one of the following would be the safest medication for this patient? A) Amitriptyline B) Duloxetine (Cymbalta) C) Escitalopram (Lexapro) D) Paroxetine (Paxil) E) Phenelzine (Nardil)
C. Escitalopram (Lexapro) is a selective serotonin reuptake inhibitor (SSRI) that is considered safe in older adults and has a lower risk of falls compared to other antidepressants listed.
43 A 30-year-old female with type 2 diabetes and obesity sees you for follow-up. She has experienced several episodes of symptomatic hypoglycemia, and because of this she stopped all of her medications except metformin (Glucophage). Her hemoglobin A1c has increased to 8.4%. Which one of the following would be the best additional treatment for this patient? A) Basal insulin (Lantus) B) Rapid-acting insulin (Humalog) C) Exenatide (Byetta) D) Glipizide (Glucotrol) E) Repaglinide
C. Exenatide is a GLP-1 receptor agonist that is not associated with hypoglycemia and can also assist with weight loss, which would be helpful in this patient with obesity. All of the other listed medications, including both types of insulin, sulfonylureas, and meglitinides, can be associated with hypoglycemia.
70 A 60-year-old male comes to your office with a 1-year history of the gradual onset of mild fatigue and dyspnea. There are no symptom triggers. He has a 20-pack-year history of cigarette smoking but stopped at age 35. An examination is significant only for a BMI of 30 kg/m2. Office spirometry reveals a decreased FVC and a normal FEV1/FVC ratio, and there are no changes after bronchodilator administration. Which one of the following would you recommend at this point? A) The 6-minute walk test B) Bronchoprovocation testing such as a methacholine challenge test C) Full pulmonary function testing D) Bronchoscopy E) A ventilation-perfusion scan
C. Family physicians are often required to manage dyspnea and evaluate common office spirometry results. The American Thoracic Society recommends full pulmonary function testing when office spirometry suggests a restrictive pattern, which is the case with this patient's normal FEV1/FVC ratio and decreased FVC.
7 During a newborn examination the patient's mother asks what she can do to decrease the risk of food allergies in her newborn son. She tells you that there is no family history of atopic dermatitis or asthma but she has a cousin with a peanut allergy. The remainder of the examination is unremarkable. You tell her that food allergy risk can be reduced by A) breastfeeding for at least 1 year B) using soy-based formula instead of cow's milk-based formula C) introducing peanut-containing foods when solids are started D) avoiding all house pets E) avoiding a day care setting
C. Food allergy affects 4%-6% of children in the United States. Immunoglobulin E (IgE)-mediated food allergy is the best understood, and symptoms can range from rhinorrhea to anaphylaxis. The two most common allergens are cow's milk and peanuts. The National Institute of Allergy and Infectious Diseases in 2017 recommended that healthy infants without known food allergy or who have mild to moderate eczema may be introduced to peanut-containing foods with other solid foods. If the parents are concerned about a reaction, introduction of peanut-containing foods may be done in the physician's office. Infants with severe eczema, egg allergy, or both should undergo peanut-specific IgE or skin-prick testing. While breastfeeding may decrease atopic disease, there is insufficient evidence that it reduces the likelihood of food allergy, and using a soy-based formula will not prevent food allergy. If there is a dog in the home there is less risk of allergy to eggs. Children who are exposed to farm animals or who attend day care are less likely to develop atopic disease. Ref: Bettcher CM, Rockwell PG, Ravikumar R: Managing food allergy in children: An evidence-based update. J Fam Pract 2020;69(7):336-343.
46 A 43-year-old female comes to your clinic for a routine health maintenance examination. She has a past medical history of diarrhea-predominant irritable bowel syndrome (IBS-D), recurrent urinary tract infections (UTIs), and bacterial vaginosis. She has no new health concerns today. She does not take any medications on a regular basis, and states that she prefers natural supplements to prescription medications. She says that she has heard that oral probiotics are beneficial and asks if they might be the right choice for her. Which one of the following is the best evidence-based approach to counseling her about oral probiotics? A) There is no evidence that they will improve her IBS B) There is no evidence that they will decrease the risk of Clostridioides (Clostridium) difficile diarrhea when she is treated for a UTI C) There is strong evidence that they will decrease the risk of antibiotic-associated diarrhea when
C. For this 43-year-old patient, there is strong evidence based on Cochrane reviews that the use of probiotics may reduce the risk of both antibiotic-associated diarrhea more generally and Clostridioides (Clostridium) difficile diarrhea specifically when antibiotics are used (level of evidence Evidence is not as strong for their impact in adults over the age of 65. The preponderance of evidence for the effective use of probiotics is with diarrhea-predominant irritable bowel syndrome, and systematic reviews have generally supported their use for this condition. There is little evidence that probiotics decrease the incidence or recurrence of urinary tract infections. Topical, not oral, preparations of probiotics have good evidence for reducing the risk of recurrent bacterial vaginosis.
68 A 3-year-old male is brought to your office by his mother because he stepped on a large wooden splinter that broke off at the surface of his left foot and since then he has been avoiding walking on that foot. On examination the bottom of the left heel is red and inflamed. Which one of the following would be most appropriate initially to visualize the splinter? A) Radiography B) Fluoroscopy C) Ultrasonography D) CT E) MRI
C. Foreign bodies can be challenging to both detect and remove. Ultrasonography is good for detecting radiolucent material such as wood or vegetation. It would be most appropriate initially to visualize the splinter.
108. In a patient with persistent respiratory symptoms, which one of the following pulmonary function abnormalities after bronchodilator administration is required for the diagnosis of COPD? A) Low residual volume B) Low total lung capacity C) An FEV1/FVC ratio <0.70 D) An FEV1 <85% of predicted E) A peak flow <90% of predicted
C. In addition to the presence of relevant factors and chronic respiratory symptoms, a postbronchodilator FEV1/FVC ratio <0.70 is required for the diagnosis of COPD. COPD is classified as mild (FEV1 >80% of predicted), moderate (FEV1 50%-79% of predicted), severe (FEV1 30%-49% of predicted), or very severe (FEV1 <30% of predicted).
33 A 72-year-old male with a history of hypertension, heart failure, and chronic kidney disease sees you for evaluation of gradually worsening lumbar pain. The pain worsens with walking but improves when he sits. He says that the pain radiates to the buttocks and down the right leg, especially with activity. He has not had any fevers, chills, or new urinary symptoms. MRI indicates severe degenerative changes resulting in moderate to severe canal stenosis at the L4-L5 level. Which one of the following would be most appropriate at this point? A) Oral diclofenac, 75 mg twice daily B) Oral pregabalin (Lyrica), 75 mg twice daily C) Physical therapy D) Referral to an orthopedic surgeon for elective surgical resolution E) Referral to a neurosurgeon for urgent surgical resolution
C. Lumbar spinal stenosis is a common cause of low back pain in older adults. Focused physical therapy has the best evidence for initial management.
72 Which one of the following types of complementary/integrative therapy is a form of mental training that requires calming of thoughts with the goal of achieving a state of detached observation? A) Aerobic exercise B) Cognitive-behavioral therapy C) Mindfulness-based meditation D) Tai chi and qi gong E) Yoga
C. Mindfulness-based meditation is a form of mental training that requires calming of thoughts with the goal of achieving a state of detached observation. Recent clinical recommendations show that aerobic and resistance exercises, yoga, and mindfulness-based meditation interventions are effective therapeutic options for depressive disorder, while both tai chi and qi gong have inconsistent effectiveness as a complementary treatment for depression.
151 An 18-year-old female comes to the urgent care clinic because of worsening nausea and vomiting, itching, and a dry cough that began about 30 minutes after she ate lunch at a nearby restaurant. She tells you that she did not experience any choking while eating her lunch, and she has not had any dysphagia, rash, or diarrhea. She takes no medications, and her past medical history is significant only for a severe nut allergy. She says that she was feeling well before today. An examination is notable only for a blood pressure of 88/60 mm Hg, mildly labored breathing, and bilateral expiratory wheezes. At this point you would administer A) albuterol (Proventil, Ventolin) B) diphenhydramine (Benadryl) C) epinephrine D) hydroxyzine (Vistaril) E) methylprednisolone (Medrol)
C. Most anaphylactic reactions occur outside of the hospital setting, and early treatment decreases both hospitalizations and mortality. This patient presents with respiratory, dermatologic, cardiovascular, and gastrointestinal symptoms, which are common in anaphylaxis. Tree nut and peanut allergies are risk factors for severe reactions. Early treatment with intramuscular epinephrine and attention to airway, breathing, and circulation are the first steps for treatment. Adjunct medications can be considered after epinephrine, but antihistamines have an onset of action of ≥1 hour and corticosteroids have an onset of action of 6 hours. Albuterol may be considered as an adjunct, but its use does not address the urgent need to resolve anaphylaxis symptoms first.
132 A 12-year-old male with type 1 diabetes is brought to your office for routine follow-up. Laboratory work performed prior to the appointment shows an LDL-cholesterol level of 120 mg/dL. In addition to counseling the patient on a heart-healthy diet and daily physical activity, which one of the following would you recommend? A) No additional measures B) Fish oil supplements C) Atorvastatin (Lipitor) D) Ezetimibe (Zetia) E) Gemfibrozil (Lopid)
C. Pediatric type 1 diabetes is recognized as a high-risk condition for the future development of cardiovascular disease. Current guidelines recommend initiating a statin, in addition to education regarding a healthy diet and physical activity, for pediatric patients in this high-risk category with an LDL-cholesterol level >100 mg/dL. Statins such as atorvastatin are recommended for first-line treatment according to multiple studies that demonstrate their efficacy and benefits in the reduction of cardiovascular morbidity and mortality, along with long-term studies demonstrating their safety. Fish oil supplements, ezetimibe, and gemfibrozil would not be appropriate recommendations for this patient at this time.
23 A 70-year-old female sees you for a Medicare annual wellness visit. She has a blood pressure of 159/90 mm Hg, a temperature of 36.7°C (98.1°F), a heart rate of 76 beats/min, a respiratory rate of 17/min, and an oxygen saturation of 98% on room air. On examination, you note a new harsh systolic murmur that is heard best at the second right intercostal space and can also be heard over the right carotid artery. A transthoracic echocardiogram reveals severe aortic stenosis. Which one of the following should you recommend for this patient? A) Antibiotic prophylaxis for dental procedures B) Transesophageal echocardiography C) Repeat echocardiography in 6 months D) Referral for aortic valve replacement
C. Severe aortic stenosis with symptoms requires referral for aortic valve replacement. Antibiotic prophylaxis is not indicated, and repeat echocardiography is not appropriate in this case.
34 A 42-year-old female presents to your office to discuss bariatric surgery and its potential complications. Her BMI is 40 kg/m2 and she has hypertension, type 2 diabetes, and osteoarthritis of both knees. If she opts to have a sleeve gastrectomy, which one of the following complications is most likely in the first 6 months? A) Cholelithiasis B) Dumping syndrome C) GERD D) Leaking at the surgical site E) Small bowel obstruction
C. Sleeve gastrectomy is currently the most common bariatric procedure. The most common complication is development of GERD, which occurs in 20% of patients.
36 An otherwise healthy 58-year-old female presents with several noduloulcerative lesions ascending up her arm from a lesion on her index finger. A fungal culture of drainage from one of the lesions reveals Sporothrix schenckii. Which one of the following would be the most appropriate initial management strategy? A) Observation for spontaneous resolution for 1 month B) Local application of heat C) Oral itraconazole (Onmel) D) Oral saturated solution of potassium iodide E) Intravenous liposomal amphotericin B (AmBisome)
C. Sporotrichosis is a skin infection caused by the Sporothrix schenckii fungus. The initial treatment strategy is oral itraconazole for 3-6 months.
27 A 20-year-old male presents with a painful second finger after his right hand was stepped on 3 days ago while he was playing basketball. A radiograph reveals a distal phalanx fracture. Which one of the following would be the most appropriate next step? A) Treat symptomatically with ice and an anti-inflammatory medication B) Tape the first and second digits together until symptoms resolve C) Splint the affected digit for 2-4 weeks D) Remove the nail to evaluate for a nail bed injury E) Refer to a hand surgeon
C. The most appropriate next step for a distal phalanx fracture (tuft fracture) is to splint the affected digit for 2-4 weeks. Symptomatic treatment with ice and anti-inflammatory medications is also appropriate.
86 A 42-year-old female sees you because of intermittent right upper abdominal pain that occurs after eating. The episodes have been gradually worsening and now last up to an hour. She has tried over-the-counter antacids, ibuprofen, and acetaminophen, which have not helped. She tells you that the last episode occurred earlier this week and the pain was so severe that it woke her up and she went to the emergency department (ED). A comprehensive metabolic panel, CBC, and lipase level performed in the ED were all normal. Right upper quadrant abdominal ultrasonography today is negative for gallstones but notable for increased echogenicity of the liver. Which one of the following would be the most appropriate next step in the evaluation? A) Plain radiography of the abdomen B) CT of the abdomen C) Hepatobiliary scintigraphy (HIDA) D) Magnetic resonance cholangiopancreatography (MRCP) E) Endoscopic retrograde cholangiopan
C. The most appropriate next step in the evaluation is: Hepatobiliary scintigraphy (HIDA) - In the presence of classic biliary symptoms and normal ultrasonography, hepatobiliary scintigraphy is the most appropriate test for functional gallbladder disease.
19 Which one of the following is needed to calculate the number needed to treat (NNT)? A) Number needed to harm B) Pretest probability C) Absolute risk reduction D) Relative risk reduction E) Likelihood ratio
C. The number needed to treat (NNT) is calculated as: 1/absolute risk reduction (ARR), where the ARR is expressed as a decimal. NNT describes the number of patients who need to receive an intervention instead of the alternative in order for one additional patient to benefit. It is different from the number needed to harm, pretest probability, absolute risk reduction, relative risk reduction, and likelihood ratio.
87 A 6-month-old male is brought to your office by his mother for a well child examination. The mother does not have any concerns. Interactions between the mother and child are appropriate and the child appears well. Which one of the following screenings is recommended at this visit? A) Autism B) Iron deficiency C) Maternal depression
C. The recommended screening at this visit is: Maternal depression - The American Academy of Pediatrics recommends formal screening for maternal depression with the Edinburgh Postnatal Depression Scale or the Patient Health Questionnaire-2 (PHQ-2) at the 1-, 2-, 4-, and 6-month well-child visits.
102 An 8-year-old female is brought to your office because of left arm pain after she fell down on the sidewalk while roller skating. She has pain, swelling, and a mild deformity of her distal forearm over the radius. Posteroanterior and lateral radiographs confirm an incomplete compression fracture of the distal radius. In addition to a short arm splint, which one of the following would be appropriate management of this fracture? A) Ultrasonography in 3 weeks B) Repeat radiography in 4 weeks C) Return to activity in 4 weeks if she is pain-free D) Follow-up and reevaluation in 6 weeks E) Referral to an orthopedist
C. This patient has a compression fracture of the distal radius, also known as a buckle fracture. It is a stable fracture, and treatment involves short-arm immobilization. Repeat imaging is unnecessary if there is no longer tenderness or pain after 4 weeks of splinting. The patient can return to full activity as tolerated without referral to an orthopedist.
47 An 86-year-old male is brought to your office by his daughter for follow-up of hypertension. His blood pressure is well controlled on amlodipine (Norvasc), 5 mg daily. His daughter is concerned about the safety of her father driving because he has become confused on several occasions. He tells you that he only drives short distances to familiar places during the daytime, and he is somewhat agitated that his daughter mentioned this topic. He has no cognitive deficits noted on a 3-item recall or a clock drawing test. Neurologic and musculoskeletal examinations reveal no deficits. Which one of the following would be the most appropriate next step in management? A) No further interventions B) Discontinuing amlodipine C) Reinforcing safe driving practices with education on age-related changes that may affect safe driving D) Recommending that he stop driving and surrender his license
C. This patient is over the age of 85 and at higher risk for a motor vehicle crash. He has shown that he has good insight and has made safety changes to his driving. Reinforcing safe driving practices would be appropriate at this time. An assessment of his driving safety, including vision and hearing evaluations, would also be appropriate. His blood pressure is well controlled, so amlodipine should be continued. Physicians can recommend that a patient stop driving, but consideration should be given to the social and emotional implications. This patient does not have any medical concerns that would necessitate a recommendation to stop driving or surrender his license.
146 A 15-year-old female is brought to your office by her parents for evaluation because they are concerned about her restrictive eating patterns and weight loss. The patient is unconcerned about these issues and says that she feels well and does not need any evaluation. On examination she has a height of 163 cm (64 in) and a weight of 43 kg (95 lb), with a BMI of 16 kg/m2. She has a pulse rate of 52 beats/min and a blood pressure of 102/68 mm Hg while seated and 84/58 mm Hg while standing. Evaluation of her teeth shows significant erosion of the enamel. When considering the psychotherapy aspect of care for this patient, which one of the following is preferred for treatment of her condition? A) Cognitive-behavioral therapy B) Dialectical behavioral therapy C) Family therapy D) Interpersonal therapy E) Psychodynamic therapy
C. This patient likely has anorexia nervosa, and family therapy is preferred for children and adolescents with anorexia nervosa. It involves parental involvement and provides consistent support for treatment goals.
42 A 34-year-old female presents with a 1-month history of increasing foot pain. She does not have any significant past musculoskeletal history, and she started a new exercise program 6 weeks ago. She has pain in the lateral side of her heel that is present both with activity and at rest. On examination you note tenderness below the lateral malleolus extending to the midfoot. Which one of the following is the most likely diagnosis? A) Calcaneal apophysitis (Sever's disease) B) Calcaneal stress fracture C) Peroneal tendinopathy D) Plantar fasciitis E) Tarsal tunnel syndrome
C. This patient most likely has peroneal tendinopathy, which is a degeneration of the peroneal tendon that involves pain or tenderness in the lateral calcaneus below the ankle along the path to the base of the fifth metatarsal. Initial treatment options include activity modification, decreasing pressure to the affected area, anti-inflammatory or analgesic medications, and eccentric exercises.
107. An 82-year-old female with atrial fibrillation treated with digoxin is started on verapamil sustained-release capsules (Calan SR) for hypertension and angina. Although she initially tolerates the medication and has a good clinical response, when you see her 1 month later she has lost 3 kg (7 lb) and reports persistent anorexia and nausea over the past 2-3 weeks. A serum chemistry profile, TSH level, and CBC are normal. Her serum digoxin level is 1.4 ng/mL (therapeutic range 0.8-1.5). Her vital signs are stable and a physical examination is notable only for rate-controlled atrial fibrillation. Which one of the following would be most appropriate at this point? A) Prescribe a therapeutic trial of an H2-blocker B) Order an upper gastrointestinal contrast study C) Withhold digoxin for several days and reinstitute at a lower dosage if necessary D) Order imaging to look for a central nervous system abnormality
C. This patient presents with typical symptoms of digitalis toxicity. When drug toxicity is suspected, the first step in management would be to discontinue or reduce the dosage of the suspected agent. In this case, the addition of verapamil will increase the serum level of digoxin, which will reach a new steady-state level in several days. Therefore, the side effects may not occur for several days while the level is increasing.
128 A 42-year-old transgender male comes to your office for a routine health maintenance examination. The patient's current medications include testosterone enanthate (Xyosted), 100 mg subcutaneously every 7 days, for gender affirmation, and medroxyprogesterone acetate (Depo-subQ Provera 104), 104 mg subcutaneously every 12 weeks, for contraception. The patient has no current chronic health conditions and no current sexual partners, but he has previously been sexually active with men and women. Which one of the following health conditions is more likely to occur in this patient compared to a female cisgender patient? A) Anemia B) Cervical cancer C) Dyslipidemia D) Kidney disease E) Venous thromboembolism
C. Transgender describes persons whose experienced or expressed gender differs from their sex assigned at birth. In the United States approximately 150,000 youth and 1.4 million adults identify as transgender, though many believe these numbers underestimate the actual prevalence. Transgender men who take testosterone may experience increased muscle mass and decreased fat mass, male pattern baldness, increased sexual desire, clitoromegaly, decreased fertility, deepening of the voice, cessation of menses, acne, and a significant increase in body hair, particularly on the face, chest, and abdomen. Risks of testosterone therapy include more atherogenic lipid profiles, an increase in blood pressure, and erythrocytosis (rather than anemia). Severe liver dysfunction is unusual at therapeutic dosages but is a concern at dosages above the recommended therapeutic range. Testosterone therapy has not been associated with cervical cancer, kidney disease, or venous thromboembolism (VTE). Estrogen-based therapies for male-to-female transgender patients do carry an increased risk for VTE. It is not clear whether increased blood pressure and dyslipidemia in these patients translate into an increase in cardiovascular events. Even so, when identified, treatment such as antihypertensive drugs and statins to address these risk factors is recommended as for any other patient.
110. A 7-year-old male is brought to the urgent care clinic with a 2-day history of fever and sore throat, with no associated cough. His temperature is 38.3°C (100.9°F) and a rapid antigen test confirms a group A -hemolytic Streptococcus infection. A prescription for penicillin is sent to the pharmacy, but the medication is never picked up due to a lack of transportation. The patient is brought to your office 2 weeks later with a fever, joint pain, shortness of breath, and chest pain. His vital signs are significant for a temperature of 38.8°C (101.8°F) and a heart rate of 118 beats/min. On examination the patient's affect is appropriate, he has a 3/6 holosystolic murmur heard best over the apex, and he has tenderness and swelling of his knees bilaterally and of his left ankle. An antistreptolysin O titer is positive, his erythrocyte sedimentation rate is 124 mm/hr (N <10), and a chest radiograph is significa
C. Using the Jones criteria for diagnosis, this patient has acute rheumatic fever, with two major criteria (carditis and polyarthritis) and two minor criteria (fever and positive erythrocyte sedimentation rate). NSAIDs such as naproxen can provide significant relief and should be administered as soon as acute rheumatic fever is diagnosed. Treatment with corticosteroids, intravenous immunoglobulins, and plasmapheresis is not considered appropriate for acute rheumatic fever but may be indicated for management of pediatric autoimmune neuropsychiatric disorders associated with streptococcal infections (PANDAS).
17 A 45-year-old female comes to your office for an annual health maintenance visit. She has a family history of type 2 diabetes in her mother and a personal history of obesity for many years. If verified with a second test, which one of the following would confirm a diagnosis of diabetes mellitus? A) A hemoglobin A1c of 6.4% B) A 2-hour plasma glucose level of 190 mg/dL on an oral glucose tolerance test C) A random glucose level of 190 mg/dL D) A fasting plasma glucose level of 130 mg/dL
D. A diagnosis of type 2 diabetes can be based on any of the following test results: a hemoglobin A1c 6.5%, a fasting plasma glucose level 126 mg/dL, a 2-hour plasma glucose level 200 mg/dL on an oral glucose tolerance test, or a random plasma glucose level 200 mg/dL with classic symptoms of hyperglycemia.
77 A 45-year-old female with a 4-year history of type 2 diabetes is taking only metformin (Glucophage) and maintaining a hemoglobin A1c of 6.6%. Her LDL-cholesterol level is 94 mg/dL. She has no complications related to diabetes and her medical history is otherwise unremarkable. Which one of the following should be added to her current medication regimen? A) A DPP-4 inhibitor B) An SGLT2 inhibitor C) A low-intensity statin D) A moderate-intensity statin E) A high-intensity statin
D. All patients between 40 and 75 years of age with diabetes mellitus and an LDL-cholesterol level 70 mg/dL should begin taking a moderate-intensity statin. It is not necessary to calculate a 10-year risk for atherosclerotic cardiovascular disease because the results do not alter the recommendation. This patient's hemoglobin A1c is <7%, which is acceptable, and she does not need additional hypoglycemic medications. She has no diabetes-specific risk-enhancing conditions such as a long duration of illness, chronic kidney disease, retinopathy , neuropathy, or an ankle-brachial index <0.9. Older age and risk-enhancing conditions may require increasing the statin to high-intensity dosages. A DPP-4 inhibitor, an SGLT2 inhibitor, and a low-intensity statin would not be appropriate for this patient at this time.
129 In a patient diagnosed with major depressive disorder, which one of the following factors should raise suspicion for a bipolar disorder? A) Diminished interest in almost all activities B) Fatigue or loss of energy C) Feelings of worthlessness or inappropriate guilt D) A parent diagnosed with a bipolar disorder E) Psychomotor retardation
D. An underlying bipolar disorder is often overlooked in patients presenting with anxiety and/or depressive symptoms. One-fourth of patients presenting with depression or anxiety have been diagnosed with a bipolar disorder. Children of parents with bipolar disorders have a 4%-15% risk of being affected compared to <2% of patients without a family history. Symptoms of diminished interest in activities, fatigue or loss of energy, feelings of worthlessness or inappropriate guilt, and psychomotor retardation are all classic symptoms of a major depressive disorder and by themselves do not raise suspicion of a bipolar disorder.
8 Which one of the following antihypertensive medications is LEAST likely to exacerbate erectile dysfunction? A) Clonidine (Catapres) B) Doxazosin (Cardura) C) Hydrochlorothiazide D) Losartan (Cozaar) E) Metoprolol
D. Angiotensin receptor blockers (ARBs) such as losartan are least likely to cause or exacerbate erectile dysfunction. ARBs may have a favorable effect on erectile dysfunction by inhibiting vasoconstriction activity of angiotensin. Clonidine, -blockers, hydrochlorothiazide, and -blockers are more likely to negatively affect erectile function. Ref: Viigimaa M, Vlachopoulos C, Lazaridis A, Doumas M: Management of erectile dysfunction in hypertension: Tips and tricks. World J Cardiol 2014;6(9):908-915. 2) Rew KT, Heidelbaugh JJ: Erectile dysfunction. Am Fam Physician 2016;94(10):820-827.
153 An 18-month-old female develops immune thrombocytopenic purpura following MMR administration in your clinic. She requires hospitalization and extensive treatment. The child's family could be eligible for compensation by the A) clinic's malpractice insurance B) vaccine manufacturer's liability coverage C) Countermeasures Injury Compensation Program D) National Vaccine Injury Compensation Program
D. Common reactions to vaccines are typically mild and include pain or swelling at the injection site, fever, drowsiness, and rash. Serious adverse reactions to vaccines are less common and in some cases rare but can include serious allergic reaction to a vaccine ingredient, febrile seizure, immune thrombocytopenic purpura, and intussusception. The National Childhood Vaccine Injury Act of 1986 established the no-fault National Vaccine Injury Compensation Program for patients and families who were injured by recommended vaccines. This law requires documentation of the manufacturer and lot number of the administered vaccine. Physicians also must document that they have provided their patients with current vaccine information statements. The program is funded by an excise tax on vaccines. Compensation for vaccine injury is not available from the clinic's malpractice insurance or the vaccine manufacturer's liability coverage. The Countermeasures Injury Compensation Program is a federal program created to provide compensation for patients who are injured or die from the administration of a countermeasure for a declared epidemic, pandemic, or national security threat. This provides coverage for emergency-use authorizations for medications and vaccines used to prevent or treat conditions such as COVID-19, Zika virus, Ebola virus, anthrax, acute radiation syndrome, and smallpox. It does not provide coverage for side effects or complications from Advisory Committee on Immunization Practices-approved routine vaccinations.
22 A 14-year-old male is brought to your office by his parents, who are concerned about his behavior. Recently he was caught shoplifting video games. He started smoking cigarettes at age 10, and he has a history of truancy from school for the past 2 years. His parents report that they have caught him starting fires outside of their home, and he often teases the family dog, whom he has injured on several occasions. The most likely diagnosis is A) antisocial personality disorder B) attention-deficit/hyperactivity disorder (ADHD) C) avoidant personality disorder D) conduct disorder E) substance abuse
D. Conduct disorder, characterized by aggression, theft, starting fires, and truancy, is the most likely diagnosis in this adolescent with concerning behavioral issues.
130 A 33-year-old female presents with palpitations and excessive sweating. A physical examination is normal. Laboratory findings include a TSH level of 0.02 U/mL (N 0.40-4.00) and a free T4 level of 3.9 ng/dL (N 0.7-1.9). Radionuclide scanning reveals no uptake. Which one of the following would explain these findings? A) Thyroid hormone resistance B) Graves disease C) A toxic nodular goiter D) Excess thyroid hormone intake E) A thyrotropin-secreting pituitary tumor
D. Excess thyroid hormone intake would cause a low TSH level with a high free T4 level. Other possibilities include an hCG-secreting tumor and the thyrotoxic phase of subacute thyroiditis. An elevated TSH level would be seen with thyroid-hormone resistance or a thyrotropin-secreting pituitary tumor. Graves disease causes a homogeneous increased thyroid uptake on radionuclide scanning, whereas a hot nodule would be expected with toxic nodular goiter.
26 A 30-year-old female presents for evaluation of chronic abdominal bloating, cramping, diarrhea, and recent weight loss. Which one of the following would be most likely to cause a false-negative result on serologic testing for celiac disease? A) A recent increase in dietary wheat consumption B) Recent use of loperamide (Imodium A-D) C) A skin rash consistent with dermatitis herpetiformis D) IgA deficiency E) Iron deficiency anemia
D. False-negative serologic results for celiac disease may occur in patients with IgA deficiency. Recent dietary wheat consumption or use of loperamide would not cause false-negative results.
104 An obese 32-year-old male is admitted to the hospital with a new onset of acute pancreatitis. A lipid panel reveals a triglyceride level of 1150 mg/dL and an HDL-cholesterol level of 30 mg/dL. Other laboratory studies are normal. His 10-year risk of atherosclerotic cardiovascular disease is <5%. His family history is positive for recurrent pancreatitis in his father and paternal grandfather. In addition to lifestyle modifications, which one of the following would be most appropriate for this patient after he is discharged? A) Atorvastatin (Lipitor) B) Colesevelam (Welchol) C) Ezetimibe (Zetia) D) Fenofibrate (Tricor) E) Omega-3-acid ethyl esters (Lovaza)
D. Fibrates reduce the likelihood and recurrence of pancreatitis due to severe hypertriglyceridemia when triglyceride levels are 500 mg/dL. This patient's risk of atherosclerotic cardiovascular disease is <7.5%, and his LDL-cholesterol level is within the normal range, so initiating a statin or ezetimibe is not indicated. Colesevelam may be used to reduce LDL-cholesterol and glucose levels but is not considered a first-line treatment. Omega-3-acid ethyl esters will reduce triglyceride levels, but this patient has severe hypertriglyceridemia, so fibrate therapy is recommended to prevent recurrent pancreatitis.
25 After a thorough history and examination, you determine that a 30-year-old male has an upper respiratory infection with a persistent cough. The best treatment for symptomatic relief of his persistent cough would be intranasal A) antibiotics B) antihistamines C) corticosteroids D) ipratropium (Atrovent) E) saline
D. For symptomatic relief of persistent cough due to upper respiratory infection, intranasal corticosteroids are the most appropriate treatment.
10 A 3-year-old male has developed multiple large areas of bullous impetigo on the legs, buttocks, and trunk after being bitten numerous times by ants. Which one of the following would be the most appropriate treatment? A) Topical mupirocin ointment B) Oral azithromycin (Zithromax) C) Oral tetracycline D) Oral trimethoprim/sulfamethoxazole (Bactrim) E) Intramuscular penicillin G benzathine (Bicillin L-A)
D. Impetigo may be caused by Streptococcus pyogenes or Staphylococcus aureus, but bullous impetigo is caused exclusively by S. aureus. Oral trimethoprim/sulfamethoxazole is an appropriate treatment for skin infections caused by S. aureus, including susceptible cases of methicillin-resistant S. aureus (MRSA). Topical mupirocin ointment is not practical in very widespread cases or in cases with large bullae. Neither azithromycin nor penicillin is a preferred treatment for impetigo, due to a high rate of treatment failure. Tetracycline should be avoided in children under 8 years of age due to a propensity to cause permanent staining of the teeth.
45 An 18-month-old female is brought to your office for a well child check. During the examination you note that she is unable to say any words. She can follow a one-step command and point to three body parts. She does wave goodbye and babble, and she appears to have normal comprehension, emotional relationships, and fine and gross motor movements. Which one of the following is the most likely diagnosis based on this patient's speech development? A) Autism spectrum disorder B) Cerebral palsy C) Congenital hearing loss D) Developmental speech and language delay E) Receptive language disorder
D. More than 90% of children can speak three words at 18 months of age and 50%-90% can speak six words. This patient scenario suggests a developmental speech delay. There is nothing in this case to suggest an autism spectrum disorder, and the normal emotional relationships are reassuring.
32 A 55-year-old female with diabetes mellitus and hypertension sees you because of a 3-month history of a persistent nonproductive cough. Two weeks after the cough began she presented to a local urgent care center with additional symptoms of sinus pressure, rhinorrhea, and subjective wheezing. A lung examination and chest radiograph performed at that visit were unremarkable. She was diagnosed with acute bronchitis and prescribed benzonatate (Tessalon). Since then, her sinus-related symptoms have abated, although her cough has not improved. Her current medications include metformin (Glucophage), lisinopril (Prinivil, Zestril), and hydrochlorothiazide, all of which were initiated 6 months ago. She has no known allergies and has never smoked. A physical examination today is unremarkable. Which one of the following is the most likely cause of her cough? A) Chronic lung disease B) Infection C) Malignancy D) A medication
D. Of the choices listed, an adverse effect of medication, specifically lisinopril, is the most likely cause of this patient's persistent cough. ACE inhibitors are among the most common causes of chronic cough.
105 A 23-year-old gravida 1 para 1 who is a single mother of a 3-day-old infant comes to your office for a newborn follow-up. She reports some sleep disturbance, mild depression without suicidal ideation, and financial concerns. Her past medical history is significant for persistent depressive disorder. The U.S. Preventive Services Task Force recommends which one of the following to help prevent perinatal depression in patients such as this? A) Exercise B) Amitriptyline C) Sertraline (Zoloft) D) Referral for cognitive-behavioral therapy
D. The U.S. Preventive Services Task Force (USPSTF) recommends counseling interventions to prevent perinatal depression in patients who are at risk. This patient has risk factors for perinatal depression, including young age, single motherhood, and a history of depression. Other risk factors include low socioeconomic status and depressive symptoms. The USPSTF found that counseling interventions, including cognitive-behavioral therapy and interpersonal therapy, were associated with a reduced risk of perinatal depression. There is insufficient evidence to assess the balance of benefits and harms of counseling interventions in patients who do not have depression but are at increased risk due to risk factors such as a history of depression.
41 A 69-year-old male sees you for a routine examination and asks about lung cancer screening. He smoked one pack of cigarettes per day for about 35 years but quit 11 years ago. According to the U.S. Preventive Services Task Force and the American College of Chest Physicians, which one of the following should you recommend? A) No screening B) An annual history and examination focusing on lung symptoms C) Annual chest radiography D) Annual low-dose chest CT
D. The U.S. Preventive Services Task Force and the American College of Chest Physicians support screening for lung cancer with annual low-dose CT in patients 50-80 years of age who have a 20-pack-year smoking history and who currently smoke or have smoked within the past 15 years. There is no evidence to support an annual history and physical examination or annual chest radiography as screening tools for lung cancer.
158 A 34-year-old male began a sexual relationship with a woman 3 months ago and the relationship ended on friendly terms last week. He received a call yesterday from the woman, who said she had developed a rash that resulted in a diagnosis of syphilis and that he should be evaluated and treated if appropriate. He has no symptoms and a serologic test for syphilis is negative. He has no known drug allergies. Which one of the following would be most appropriate at this time? A) Daily self-inspection of the penis to identify a chancre B) Observation with a repeat serologic test for syphilis in 6 weeks C) Azithromycin (Zithromax), 2 g orally as a single dose D) Penicillin G benzathine (Bicillin L-A), 2.4 million units intramuscularly as a single dose E) Penicillin G benzathine, 2.4 million units intramuscularly once weekly for 3 weeks
D. The evidence suggests that this patient did not have syphilis prior to this lone contact and a diagnosis of syphilis cannot be confirmed by examination or testing at this point. He should be treated presumptively for early syphilis, even though the serologic test result is negative, because he had sexual contact within the past 90 days with a person who was diagnosed with secondary syphilis. The same is true for individuals exposed to sex partners diagnosed with primary or early latent syphilis during the same time period. When the contact occurred more than 90 days before confirmation of a negative serologic test result, no treatment is necessary. The recommended treatment for individuals such as this patient and for those with primary or secondary syphilis is a single dose of penicillin G benzathine, 2.4 million units. For patients with a penicillin allergy, oral treatment with doxycycline, 100 mg twice daily; tetracycline, 500 mg four times daily; or azithromycin, 2 g as a single dose, has been effective as an alternate treatment option but should only be used when penicillin is contraindicated and should be followed by close monitoring of serologic tests.
133 You are reviewing and updating your routine health care examination electronic health record templates to include formal recommendations from the U.S. Preventive Services Task Force. You also consider age-specific causes of mortality to create corresponding preventive strategies. Which one of the following is the leading cause of mortality among people 45-64 years of age? A) Accidents B) Diabetes mellitus C) Heart disease D) Malignancy E) Suicide
D. The leading cause of mortality among people aged 45-64 years is malignancy. The U.S. Preventive Services Task Force generally recommends a focus on cancer screening in this age group. Accidents are the third most common cause of mortality in people 45-64 years of age, but they are the leading cause of mortality among people 15-44 years of age, and preventive recommendations reflect interventions to prevent accidents. Diabetes mellitus is the sixth most common cause of mortality in people 45-54 years of age and the fifth most common cause in people 55-64 years of age. Heart disease is the second most common cause of mortality in people 45-64 years of age, but it is the leading cause of mortality in people 65 years of age and older. Suicide is the fourth most common cause of mortality in adults 45-54 years of age and the eighth most common cause in adults 55-64 years of age.
85 A 55-year-old male presents to your office to establish care. He has a history of hypertension, hypercholesterolemia, and coronary artery disease. He had a non-ST-elevation myocardial infarction (NSTEMI) 3 years ago. An echocardiogram at that time was normal and he received a single drug-eluting stent. He has not seen a cardiologist since then and would prefer not to see one unless it is necessary, due to his high insurance copayments for specialist visits. His current medications include clopidogrel (Plavix), aspirin, atorvastatin (Lipitor), lisinopril (Prinivil, Zestril), and carvedilol (Coreg). A review of systems is negative. His vital signs include a blood pressure of 120/72 mm Hg, a heart rate of 80 beats/min, and a respiratory rate of 12/min. A physical examination is unremarkable, a basic metabolic panel is normal, and his hemoglobin A1c is 5.7%. His LDL-cholesterol level is 60 mg/dL, his HDL-cholesterol
D. The most appropriate medication to discontinue in this patient is: Clopidogrel - For patients with stable coronary artery disease, continued control of blood pressure and cholesterol is crucial. Based on guidelines, it would be reasonable for this patient to stop dual antiplatelet therapy at this time by discontinuing clopidogrel.
81 A 68-year-old female comes to your office for a follow-up visit for diabetes mellitus. Her home glucose monitor record shows a range of 68-125 mg/dL. Her medications include atorvastatin (Lipitor), 40 mg daily; metformin (Glucophage), 750 mg twice daily; and insulin glargine (Lantus), 10 U nightly. Laboratory studies are remarkable for a hemoglobin A1c of 5.8% and a creatinine level of 0.98 mg/dL (N 0.6-1.1). She maintains healthy lifestyle behaviors such as walking 30 minutes 5 days per week and avoiding sweetened beverages. Which one of the following would be the most appropriate treatment plan? A) Continue the current medication regimen B) Increase the insulin glargine dosage C) Increase the metformin dosage D) Discontinue insulin glargine E) Discontinue metformin
D. The most appropriate treatment plan is: Discontinue insulin glargine - This patient has a well-controlled hemoglobin A1c without evidence of significant hyperglycemia, and the insulin glargine could be discontinued to simplify her regimen.
89 A 65-year-old female sees you because of increased irritability and confusion. She has a history of major depression, essential hypertension, and type 2 diabetes. Her medications include sertraline (Zoloft), 100 mg daily; lisinopril (Prinivil, Zestril), 20 mg daily; and metformin (Glucophage), 500 mg twice daily. She recently sustained a right distal radius fracture as a result of a fall, and she has been taking tramadol, 50 mg, every 6 hours for pain control. On examination the patient has a temperature of 38.2°C (100.8°F), a pulse rate of 96 beats/min, a respiratory rate of 16/min, and a blood pressure of 124/78 mm Hg. She appears confused and you note a bilateral tremor in the upper extremities, brisk reflexes, and two beats of clonus on a bilateral foot examination. The examination is otherwise normal. A CBC, comprehensive metabolic panel, chest radiograph, and CT of the head are all within normal limits. Wh
D. The most likely cause of this patient's symptoms is: Serotonin syndrome - This patient exhibits symptoms consistent with serotonin syndrome, likely due to the concurrent use of sertraline and tramadol.
152 A premenopausal 48-year-old female comes to your office because of a 3-month history of increased frequency and volume of vaginal bleeding. She typically menstruates every 24 days but now has bleeding every 10-14 days and notes an increased volume of blood as well. She feels well otherwise. Her cervical cancer screening is current. A bimanual examination reveals blood at the cervical os. Otherwise, the cervix appears normal, the uterus is normal to palpation, and there are no adnexal masses. Which one of the following should be the next step in the evaluation? A) Reassurance that irregular menses are common in the perimenopausal period B) HPV testing C) CT of the pelvis with and without contrast D) An endometrial biopsy
D. This patient has abnormal uterine bleeding characterized by an increased frequency and volume of vaginal bleeding. Due to the increased risk of endometrial cancer, current guidelines recommend that all women >45 years of age presenting with abnormal uterine bleeding undergo endometrial sampling. Irregular menses can occur during the perimenopausal period but this patient's increased frequency and volume of vaginal bleeding combined with her age warrant further evaluation. In a patient with an up-to-date Papanicolaou smear and normal-appearing cervix, HPV testing would have no role in the evaluation. Transvaginal ultrasonography is recommended if a bimanual examination is abnormal or if symptoms persist despite treatment. CT is rarely indicated if imaging is necessary because transvaginal ultrasonography is preferred.
40 A 28-year-old female presents with a depressed mood and sleep disturbance. She tells you that this has occurred for the past 4 years but only during the winter months. Her past medical history and a physical examination are unremarkable. Which one of the following interventions has the strongest evidence for preventing recurrence of her condition? A) Exercise B) Light therapy C) Cognitive-behavioral therapy D) Bupropion (Wellbutrin XL) E) Fluoxetine (Prozac)
D. This patient has seasonal affective disorder (SAD) that has recurred. Bupropion is the only medication beneficial for prevention of SAD. Light therapy and SSRIs are helpful for treating this disorder but do not prevent it. Exercise and cognitive-behavioral therapy are beneficial adjuncts to treatment but would not prevent recurrence.
16 A 24-year-old female comes to your office with a 1-day history of the gradual onset of nonradiating worsening right lower quadrant abdominal pain, nausea, emesis, and chills. Her symptoms are minimally relieved with ibuprofen, 400 mg. Her menstrual period 2 weeks ago was normal. She has not had any dysuria, vaginal discharge, or change in bowel habits. Her past medical and surgical histories are notable only for dysmenorrhea managed with an anti-inflammatory medication as needed. An examination reveals a temperature of 38.1°C (100.6°F), discomfort with position change, right lower quadrant abdominal tenderness and guarding, and diminished bowel sounds. Point-of-care testing shows an elevated WBC count, and a urinalysis and pregnancy test are negative. Which one of the following would you recommend at this time? A) A plain film of the abdomen B) Abdominal ultrasonography C) Pelvic ultrasonography with transvagina
D. This patient has suspected appendicitis, and CT of the abdomen and pelvis with intravenous contrast is the preferred initial imaging study. Ultrasonography is preferred in children, but not adults, as the initial study for suspected appendicitis.
115 An exclusively breastfed 2-month-old male is brought to your office by his mother for a routine well child visit. His mother indicates that the infant received all recommended care at birth, is feeding well, and is meeting expected developmental milestones. Growth parameters are consistent with previously charted data. A physical examination is unremarkable except for bruising of the right lateral arm and left anterior thigh. When questioned about these findings, the mother states that she is not overly concerned, as he is "quite an active little kicker" when he is lying in his crib. She says that his 4-year-old and 6-year-old brothers are both physically active and bruise easily as well. Which one of the following would be most appropriate at this point? A) Reassurance B) A serum vitamin D level C) A serum vitamin K level D) Abdominal ultrasonography E) Immediate referral for a child abuse investigation
E. Child abuse is a common problem that often goes unrecognized until serious injury or death occurs. As many as 20% of child homicide victims have contact with a healthcare provider within a month of their death; thus, family physicians are ideally positioned to identify and intervene in suspected cases of child abuse. Furthermore, physicians are mandatory reporters of suspected child abuse and neglect to the appropriate child protective service and/or law enforcement agency, according to the provisions and procedures of their state of practice. Any bruising in a nonmobile infant, particularly under 4 months of age, warrants further evaluation for physical child abuse (SOR C). For the 2-month-old in this case, the presence of bruising on the arm and thigh should raise concern and prompt immediate referral for a child abuse investigation. For children under 2 years of age with suspected abuse, skeletal survey imaging is also recommended (SOR C).
6 A 57-year-old female with diabetes mellitus comes to your office for a routine follow-up. Her current medications include metformin (Glucophage), 1000 mg twice daily. She tells you that she does not exercise regularly and finds it difficult to follow a healthy diet. A hemoglobin A1c today is 7.5%. She does not want to add medications at this time, but she does want to get her hemoglobin A1c below 7%, which is the goal that was previously discussed. Which one of the following would be the most effective way to improve glucose control for this patient? A) Discuss the components of a healthy diabetic diet and encourage her to follow it more closely B) Discuss the importance of regular exercise and encourage her to exercise 30-45 minutes daily C) Recommend that she check her glucose level 1-3 times daily to help determine what adjustments need to be made D) Start her on an additional medication E) Refer her to a di
E. Counseling by a diabetic educator or team of educators for medical nutrition therapy lowers hemoglobin A1c by 0.2-0.8 percentage points in patients with type 2 diabetes. While a healthy diabetic diet and regular exercise are important, simply reminding the patient of that fact is not likely to be as successful as comprehensive diabetic education. According to the Society of General Internal Medicine in the Choosing Wisely campaign, patients with type 2 diabetes who are not on insulin therapy should not check their blood glucose level daily. An additional medication will likely decrease the hemoglobin A1c, but this patient has expressed a desire to avoid additional medication, is near goal, and is not currently managing her diabetes with adequate lifestyle changes, so it would be appropriate to respect her wishes and pursue proven interventions that do not require medication. Ref: Lyon C, Fields H, Langner S, DeSanto K: Diabetes education and glycemic control. Am Fam Physician 2018;97(4):269-270.
144 A patient begins to cry when you tell her that the mammogram she had yesterday shows an abnormality requiring further imaging. The most appropriate response at this time is to A) tell her there is no need to cry B) quickly reassure her that this is most likely benign C) reassure her that most breast cancers have a long survival rate D) wait to specifically address her emotional response until after you have given her more complete information E) stop giving information and address her apparent sadness prior to continuing
E. Delivering life-altering news requires sensitivity. It's important to acknowledge the patient's emotions before providing further information or reassurance.
35 A 14-year-old female is brought to your office for a well child check and a sports physical examination. During the substance abuse screening she says that she does not drink alcohol or smoke marijuana or traditional cigarettes, but occasionally uses e-cigarettes with her friends. Which one of the following statements describing e-cigarettes is true? A) E-cigarettes do not contain heavy metals B) The nicotine in e-cigarettes is not addictive C) Teens who use e-cigarettes are less likely to use marijuana D) More teens use traditional cigarettes than e-cigarettes E) E-cigarette use among teens increases the likelihood of cigarette smoking
E. E-cigarette use has become popular among youth, and teens who use e-cigarettes are more likely to use traditional cigarettes in the future.
73 To determine compliance with prescribed medications and detect use of illicit substances, your clinic uses urine drug screening with an immunoassay qualitative point-of-care test to monitor patients who are on long-term opioid therapy. Which one of the following is most likely to result in a false-negative result and require confirmatory testing for detection? A) Cannabis B) Cocaine C) Codeine D) Morphine E) Oxycodone (OxyContin)
E. Immunoassay drug screenings can be performed at the point of care and are relatively inexpensive. Typical immunoassays can detect nonsynthetic opioids such as morphine and codeine, as well as illicit substances such as amphetamines, cannabinoids, cocaine, and phencyclidine. However, these immunoassays do not reliably detect synthetic or semisynthetic opioids such as oxycodone, oxymorphone, methadone, buprenorphine, and fentanyl, as well as many benzodiazepines. Confirmatory testing is needed in situations with an unexpected negative result to distinguish a false negative from a true negative.
20 A 30-year-old male comes to your office for evaluation of hand weakness. On examination, you detect weakness when he tries to bring his thumb and index finger together. For confirmation, you ask him to try to hold on to a piece of paper between his thumb and index finger while you try to pull it away. He is unable to resist when you pull on the paper. The most likely explanation for these findings is an injury to the A) brachial plexus B) median nerve C) musculocutaneous nerve D) radial nerve E) ulnar nerve
E. Initial general neurovascular assessment of an upper extremity injury includes evaluating for radial pulse and digit movement and sensation. Weakness of the thumb and index finger pincer mechanism is indicative of an ulnar nerve injury. Symptoms related to the median nerve generally include paresthesia of the thumb, index finger, and long finger.
58 A 27-year-old soccer player presents with anterior hip pain along with a clicking sensation in the hip when he runs or attempts lateral movements. The flexion, adduction, and internal rotation (FADIR) test and the flexion, abduction, and external rotation (FABER) test both elicit pain. You suspect a labral tear. The most accurate imaging test for the suspected diagnosis is A) a standing radiograph of the hip B) anteroposterior and frog-leg lateral radiographs of the hip C) CT of the hip D) a bone scan E) MR arthrography
E. MR arthrography is the most accurate imaging test for diagnosing a labral tear in the hip.
51 Which one of the following has been shown to be an appropriate therapeutic intervention for nonspecific low back pain? A) Bed rest B) A lumbar brace C) Muscle relaxants D) Shoe insoles E) Yoga
E. Nonspecific low back pain is best managed with physical activity, including core strengthening, physical therapy, or yoga. Bed rest, lumbar braces, muscle relaxants, and shoe insoles have shown little benefit.
79 A 45-year-old female presents with a lesion on her mid-back (shown below) that measures 1.2 mm in diameter. A punch biopsy confirms nodular basal cell carcinoma. She is otherwise healthy and does not take any daily medications. She is concerned about the cosmetic appearance after treatment. Which one of the following would be the most appropriate treatment strategy? A) No further management B) Topical fluorouracil 5% (Efudex) C) Cryotherapy D) Curettage and electrodesiccation E) Standard excision with 4-mm margins
E. Surgical excision is indicated for the management of larger basal cell carcinomas. Although this patient had a punch biopsy, that is not considered curative, and excision with wide margins is indicated. Topical therapy and cryotherapy are reserved for patients who decline surgery or for cases in which surgery is contraindicated. The combination of curettage and electrodesiccation is a management option, but the cosmetic results are not as desirable as with excision.
69 A 55-year-old male sees you because of a second flare of gout. He has also had an elevated blood pressure at the last few visits to your clinic and is hypertensive again today. In addition to treating his gout flare, which one of the following would be the most appropriate agent to treat his hypertension in light of his presenting problem? A) Atenolol (Tenormin) B) Hydralazine C) Hydrochlorothiazide D) Lisinopril (Prinivil, Zestril) E) Losartan (Cozaar)
E. The 2020 American College of Rheumatology guideline for the management of gout generated numerous recommendations, including the management of concurrent medications in patients with gout. In such patients, losartan is the preferred antihypertensive agent when possible. Hydrochlorothiazide should typically be changed to another agent, such as losartan, when feasible in patients with gout.
38 A 72-year-old female with a history of type 2 diabetes presents with a 4-month history of a burning sensation along her tongue. Her diabetes is currently well controlled with metformin (Glucophage). Her other medications include atorvastatin (Lipitor) and lisinopril (Prinivil, Zestril). Her past medical history is otherwise unremarkable. An examination reveals a smooth, glossy, erythematous tongue. A laboratory evaluation should include which one of the following tests? A) Angiotensin converting enzyme B) Antineutrophil cytoplasmic antibody C) Serotonin D) TSH E) Vitamin B12
E. The history and physical examination are consistent with atrophic glossitis. Routine vitamin B12 monitoring or supplementation is appropriate for patients who take metformin chronically.
88 A 42-year-old female presents to your office with heavy menstrual periods and pelvic pressure. Her symptoms began several years ago and have gradually worsened. Laboratory findings are notable for a mild microcytic anemia. Pelvic ultrasonography identifies a 7-cm submucosal mass. She wants to avoid a hysterectomy but desires a treatment that will provide symptom relief, decrease the volume of the mass, and have a sustained effect. Which one of the following would be most appropriate for this patient? A) Expectant management B) A GnRH agonist C) A selective estrogen receptor modulator D) A levonorgestrel-releasing IUD (Mirena) E) Uterine artery embolization and occlusion
E. The most appropriate management for this patient is: Uterine artery embolization and occlusion - Evidence supports the effectiveness of uterine artery embolization and occlusion for reducing fibroid size and providing symptom relief, with lasting effects up to 5 years.
83 A 78-year-old male with terminal lung cancer and long-standing COPD is admitted to a regular medical-surgical care unit pending transfer to the hospice unit within the next day. You are called about worsening anxiety and dyspnea. The patient is alert and anxious. He has a blood pressure of 150/94 mm Hg, a pulse rate of 96 beats/min, a respiratory rate of 24/min, and an oxygen saturation of 93% on 2 L/min of oxygen via nasal cannula. Which one of the following would be most effective in this situation? A) 40% oxygen by venti-mask B) Dexamethasone C) Hyoscyamine (Anaspaz) D) Lorazepam (Ativan) E) Morphine sulfate
E. The most effective intervention in this situation is: Morphine sulfate - Opiates, such as morphine sulfate, are the most effective agents for treating dyspnea and anxiety in patients with terminal cancer.
37 During a quality improvement project you notice significant variations in the ordering patterns of physicians in your group when checking for proteinuria. Some physicians routinely order spot protein/creatinine ratios while others order spot albumin/creatinine ratios. When comparing these two options, one advantage of spot albumin/creatinine ratios is that they A) more convenient for the patient B) unaffected by exercise C) unaffected by menstruation D) elevated only in patients with diabetes mellitus E) able to detect lower levels of proteinuria
E. The spot albumin/creatinine ratio is able to detect lower levels of proteinuria as compared to the spot protein/creatinine ratio.
120 A 69-year-old male presents with a several-week history of difficulty swallowing that has gradually worsened. At first he noted trouble with passing larger boluses of food through his mid-chest area. Now he states that even ice cream is a problem. He has a past history of GERD that he has treated with omeprazole (Prilosec) intermittently over the past several years. He also takes enteric-coated aspirin, 81 mg daily, for his heart. There is no history of tobacco use. He drinks about six beers a week. On examination you note a 5-kg (11-lb) weight loss over the past 3 months but the remainder of the examination is normal. Which one of the following would be the most appropriate next step in the evaluation? A) A trial of omeprazole, 40 mg daily, for 8 weeks B) Chest CT with and without contrast C) Barium esophagography D) High-resolution esophageal manometry E) Esophagogastroduodenoscopy
E. This patient has several risk factors for a significant problem such as esophageal cancer, including age over 50, weight loss, and progressive symptoms. Esophagogastroduodenoscopy (EGD) is needed without delay. This approach would allow biopsy of any lesions seen and therapeutic dilatation if a benign-appearing stricture is noted. Biopsies are also needed to diagnose eosinophilic esophagitis. If the EGD does not identify a problem, further workup should then proceed. CT may identify a source of extrinsic pressure. Barium esophagography may detect mild narrowing or esophageal webs missed on EGD. Esophageal motility disorders may be diagnosed with esophageal manometry. Waiting 8 weeks to see if his symptoms improve with regular use of a proton pump inhibitor would not be appropriate in this patient with symptoms that are worrisome for esophageal cancer.
61 A 56-year-old male who has heart failure with reduced ejection fraction sees you for follow-up. He is stable but over the past year has noted an increase in dyspnea with moderate activity. His blood pressure is well controlled today. His current medications include carvedilol (Coreg), losartan (Cozaar), and escitalopram (Lexapro). Which one of the following additions to his current medication regimen has the best evidence for reducing his risk of mortality from heart failure? A) Aspirin B) Atorvastatin (Lipitor) C) Furosemide (Lasix) D) Hydrochlorothiazide E) Spironolactone (Aldactone)
E. This patient has symptomatic New York Heart Association class II heart failure, and an escalation in therapy is warranted. Both -blockers and aldosterone antagonists have been shown to reduce mortality in patients with symptomatic heart failure. Of the medications listed (aspirin, atorvastatin, furosemide, hydrochlorothiazide, and spironolactone), spironolactone is the best choice to reduce heart failure-related mortality.
124 A 45-year-old male presents to your office with intermittent chest pain for the past few days, although he is currently pain-free after taking aspirin at home. He tells you that while running this morning he had pain every time he ran uphill. The pain is a dull ache on his left chest wall. He has no other associated symptoms and no significant past medical history or family history. His vital signs are stable and a physical examination is unremarkable. An EKG performed at this visit is shown below, along with a previous EKG. Which one of the following would be most appropriate at this point? A) An exercise stress test B) Stress echocardiography C) Coronary CT angiography D) Referral to a cardiologist E) Transportation via ambulance to the emergency department
E. This patient presents with risk factors for coronary artery disease, including male sex and activity-related chest pain. He also has a new left bundle branch block, which necessitates a trip to the emergency department for urgent evaluation. If there were no EKG changes, the patient would be at moderate risk for acute coronary syndrome, and further evaluation with an exercise stress test, stress echocardiography, or coronary CT angiography would be indicated. Referral to a cardiologist would lead to further delay and would not be appropriate.
135 A 61yo F comes for an annual visit. She has lab results from a blood donation. She donated blood for the first time in several years but was informed afterward that she is ineligible and should follow up with her primary care physician. She is asymptomatic. She does not recall receiving the hepatitis B immunization series. The letter she received included the following hepatitis B test results: Anti-HBcscreeningtest - positive HBsAgscreeningtest - negative HBVnucleicacidtest(NAT) - negative You order additional hepatitis B testing, with the following notable results: Anti-HBs - 56mIU/mL(N<10) Anti-HBcIgM - nonreactive Which one of the following is the most likely explanation? A) These results indicate a false positive B) She was never infected with hepatitis B C) She has an acute hepatitis B infection D) She has a chronic hepatitis B infection E) She has recovered from a past hepatitis B infection
E. This patient's laboratory studies are consistent with a past natural hepatitis B virus (HBV) infection, and she is now immune. If she had never been infected, her anti-HBc and anti-HBs would both be negative. If she had an acute infection, the HBsAg, anti-HBc IgM, and HBV nucleic acid test (NAT) would have all been positive along with the total anti-HBc. If she had a chronic infection, the HBsAg and HBV NAT would be positive in addition to the total anti-HBc. Furthermore, the anti-HBs would be negative in both acute and chronic infection since its presence is associated with recovery from infection. If her anti-HBc screening test were a false positive, the anti-HBs would be negative.
140 A 42yo F who owns a bakery presents with a several-month history of gradually worsening pain, swelling, and paresthesia affecting her entire right arm whenever she has to blend ingredients by hand. She says that her "arm veins pop out" and her arm develops a deep aching pain if she has to stir for very long. Due to the pain and swelling, she is no longer able to make wedding cakes and is concerned about her business. She denies trauma and has no swelling in her L arm. She has a history of HTN, on losartan (Cozaar). On examination, the patient has full active range of motion, and the Neer and Hawkins impingement tests of the shoulder are negative. Based on this patient's history and the physical examination findings, which one of the following is the most likely diagnosis? A) Carpal tunnel syndrome B) Cervical radiculopathy C) Complex regional pain syndrome D) Raynaud's disease E) Thoracic outlet syndrome
E. Thoracic outlet syndrome can be differentiated into neurogenic, venous, or arterial, with neurogenic being the most common, constituting more than 95% of cases. This patient has venous thoracic outlet syndrome, which is the second most common, occurring in about 3% of cases. Swelling of the arm with associated pain strongly suggests obstruction of the subclavian vein. Paresthesias in the fingers and hand are common, likely due to swelling rather than nerve compression at the thoracic outlet. Venous thoracic outlet syndrome is easily identified by swelling, cyanosis, and distention of superficial veins in the arm. Due to the exceptionally high risk of developing a venous thrombosis, patients should undergo diagnostic evaluation with upper extremity venous duplex ultrasonography. False negatives are common in patients without a thrombus, and in such cases, the patient may benefit from evaluation with either contrast-enhanced upper extremity CT or magnetic resonance venography. If a thrombosis is present, anticoagulation should be started immediately and catheterization of the vein should be performed with thrombolysis with or without balloon angioplasty. Ultimately, the patient will require surgical decompression.
75 A 60-year-old female with a BMI of 24 kg/m2 presents with bilateral knee pain that is greater in the left knee. She has no morning stiffness but the pain sometimes prevents her from completing normal activities such as grocery shopping. You perform an evaluation and diagnose osteoarthritis of the knee. She does not want to take prescription medications and asks you if supplements or other treatments would be helpful. Which one of the following would you recommend? A) Lateral wedge insoles B) Vitamin D supplements C) Glucosamine and chondroitin supplements D) Hyaluronic acid injections E) Physical therapy
E. Treatments with evidence of effectiveness for knee osteoarthritis include exercise, physical therapy, knee taping, and tai chi. Medical treatments should begin with full-strength acetaminophen and topical therapy, then NSAIDs and, selectively, tramadol or other opioids. Lateral wedge insoles, vitamin D supplements, glucosamine and chondroitin supplements, and hyaluronic acid injections are all ineffective. According to the Choosing Wisely campaign from the American Academy of Orthopaedic Surgeons, glucosamine and chondroitin should not be used in knee osteoarthritis, and lateral wedge insoles should not be used for medial knee osteoarthritis.
80 A 23-year-old primigravida comes to your office for her initial obstetric visit. She is at 13 weeks gestation based on the dates of her last menstrual period. She is a nonsmoker and does not drink alcohol or use illicit substances. Her vital signs are remarkable for a blood pressure of 142/92 mm Hg and a BMI of 32 kg/m2. She says that she has been diagnosed with hypertension in the past but has not taken any medications for it. In addition to a prenatal vitamin, which one of the following would you recommend for her? A) No additional medications B) Aspirin C) Ferrous sulfate D) Folic acid E) Labetalol (Trandate)
In this situation, you would recommend: Labetalol (Trandate) - Given her elevated blood pressure and a history of hypertension, initiation of antihypertensive medication is indicated.